ĐỀ ÔN TẬP HSG HOÁ 10 V2

You might also like

Download as pdf or txt
Download as pdf or txt
You are on page 1of 63

TRƯỜNG THPT CHUYÊN HÀ NỘI - AMSTERDAM ĐỀ THI MÔN HÓA HỌC KHỐI 10

ĐỀ THI ĐỀ XUẤT NĂM 2022


Thời gian làm bài 180 phút
(Đề này có 05 trang, gồm 10 câu)

Câu 1. (2,0 điểm) Cấu tạo nguyên tử, phân tử, định luật tuần hoàn
1. Khi phóng tia lửa điện qua các nguyên tử hiđro ở áp suất thấp, các electron bị kích thích lên trạng thái năng
lượng cao hơn. Sau đó, electron nhanh chóng chuyển về mức năng lượng cơ bản (n=1) và bức xạ ra photon
với các bước sóng khác nhau tạo thành dãy phổ. Tính bước sóng (λ) nhỏ nhất và bước sóng lớn nhất theo nm
của dãy phổ nếu electron chuyển từ n > 1 về n = 1. Biết trong hệ một electron, một hạt nhân, năng lượng của
electron được tính theo công thức:
Z2
En = - 13,6. 2 (eV). Cho: h = 6,626.10-34 J.s ; c = 3.108 m/s; 1eV= 1,6.10-19J
n
2. Nguyên tố 𝑋 có electron cuối cùng ứng với 4 số lượng tử có tổng đại số bằng 2,5. Xác định
nguyên tố 𝑋, viết cấu hình electron và cho biết vị trí của 𝑋 trong bảng tuần hoàn?
Câu 2. (2,0 điểm) Tinh thể
1. Trong máu cơ thể người có màu đỏ vì chứa hemoglobin (chất vận chuyển sắt). Máu của một số động
nhuyễn thể không có màu đỏ vì chứa một kim loại khác (X). Tế bào đơn vị (ô mạng cơ sở) lập phương tâm
diện của tinh thể X, có cạnh 3,62.10-8 cm. Khối lượng riêng của nguyên tử này là 8920 kg/m3.
a) Tính thể tích các nguyên tử trong một tế bào và phần trăm thể tích của tế bào bị chiếm chỗ bởi các nguyên
tử.
b) Xác định nguyên tố X.
2. Mạng lưới tinh thể KBr có dạng lập phương tâm diện với thông số mạng a = 6,56A0. Hãy tính khối lượng
riêng của tinh thể KBr. Cho MK = 39; MBr = 79,9; Số Avogadro𝑁𝐴 = 6,023.1023.
Câu 3. (2,0 điểm) Phản ứng hạt nhân
1. 32
P phân rã β- với chu kì bán huỷ là 14,28 ngày, được điều chế bằng phản ứng giữa nơtron với hạt nhân 32

S.
a) Viết các phương trình phản ứng hạt nhân để điều chế 32
P và biểu diễn sự phân rã phóng xạ 32
P.
b) Có hai mẫu phóng xạ 32
P được kí hiệu là mẫu I và mẫu II. Mẫu I có hoạt độ phóng xạ 20 μCi được lưu
giữ trong bình đặt tại buồng làm mát có nhiệt độ 10°C. Mẫu II có hoạt độ phóng xạ 2 μCi bắt đầu được lưu
giữ cùng thời điểm với mẫu I nhưng ở nhiệt độ 20°C. Khi hoạt độ phóng xạ của mẫu II chỉ còn 5.10 -1 μCi thì
lượng lưu huỳnh xuất hiện trong bình chứa mẫu I là bao nhiêu gam? Trước khi lưu giữ trong bình không có
lưu huỳnh. Cho: 1Ci = 3,7.1010 Bq (1 Bq = 1 phân rã/giây); số Avogadro
NA = 6,02.1023 mol-1; hoạt độ phóng xạ A = 𝜆N (𝜆 là hằng số tốc độ phân rã, N là số hạt nhân phóng xạ ở thời
điểm t).

1
2. a) 238
U tự phân rã liên tục thành một đồng vị bền của chì. Tổng cộng có 8 hạt α được phóng ra trong quá
trình đó. Hãy giải thích và viết phương trình phản ứng chung của quá trình này.
b) Uran có cấu hình electron [Rn]5𝑓 3 6𝑑1 7𝑠 2 . Nguyên tử này có bao nhiêu electron độc thân? Có thể có
mức oxi hoá cao nhất là bao nhiêu?
c) UF6 là chất lỏng dễ bay hơi được ứng dụng phổ biến để tách các đồng vị uran. Hãy viết phương trình
phản ứng có UF6 được tạo thành khi cho UF4 tác dụng với ClF3.
Câu 4. (2,0 điểm) Nhiệt hóa học
Cho phản ứng hóa học:
2𝐻2 𝑂(𝑙) → 2𝐻2 (𝑘) + 𝑂2 (𝑘) (1)
𝐶(gr) + 𝑂2 (𝑘) → CO2 (𝑘) (2)
2𝐶2 𝐻6 𝑂2 (𝑙) + 5𝑂2 (𝑘) → 4CO2 (𝑘) + 6𝐻2 𝑂(𝑙) (3)
Ở 300K ta có:
Phản ứng ΔH0 (kJ) ΔG0 (kJ)
(1) 571,155 473,928
(2) -393,129 -394,007
(3) -2286,293 -2353,089
Còn có:
Chất C (gr) H2 (k) O2 (k) C2H6 (l)
Cp (J.K-1.mol-1) 8,527 28,591 29,176 148,181
Nhiệt dung trong bình đó hằng định trong khoảng từ 280K đến 370K.
Với phản ứng:
2𝐶(gr) + 3𝐻2 (𝑘) + 𝑂2 (𝑘) ⇄ 𝐶2 𝐻6 𝑂2 (𝑙) (4)
Hãy:
a) TínhΔU04 và ΔG04 tại 300K.
b) Tìm phương trình biểu thị ΔH04 là hàm của nhiệt độ và cho biết phương trình này được áp dụng trong khoảng
nhiệt độ nào?
Câu 5. (2,0 điểm) Cân bằng hoá học trong pha khí
Khi đun nóng đến nhiệt độ cao PCl5; bị phân huỷ theo phương trình:

PCl5 (𝑘) ⇄ PCl3 (𝑘) + Cl2 (𝑘)


1. Cho m gam PCl5 vào một bình dung tích V, đun nóng bình đến nhiệt độ T (K) để xảy ra phản ứng phân li
PCl5. Sau khi đạt tới cân bằng áp suất trong bình bằng P. Hãy thiết lập biểu thức Kp theo độ phân li a và áp
suất P. Thiết lập biểu thức Kp theo 𝛼, m và V.
2. Trong thí nghiệm thực hiện ở nhiệt độ T, người ta cho 83,3 gam PCl5 vào bình dung tích V1. Sau khi đạt
trạng thái cân bằng đo được P = 2,700 atm. Hỗn hợp khí trong bình có tỉ khối so với hiđro bằng 68,826. Tính
𝛼 và Kp.

2
3. Trong thí nghiệm 2 giữ nguyên lượng PCl5 và nhiệt độ như ở thí nghiệm V1 nhưng thay dung tích là V2 thì
đo được áp suất cân bằng là 0,500 atm. Tính tỉ số 𝑉2 /𝑉1 .
4. Trong thí nghiệm 3 giữ nguyên lượng PCl5 và dung tích bình V1 như ở thí nghiệm 1 nhưng hạ nhiệt độ của
bình tới 𝑇3 = 0,9𝑇1 thì đo được áp suất cân bằng là 1,944 atm. Tính Kp và 𝛼. Từ đó cho biết phản ứng phân
li PCl5 thu nhiệt hay tỏa nhiệt. Cho 𝐶𝑙 = 35,453; 𝑃 = 30,974; 𝐻 = 1,008; các khí đều là khí lí tưởng.
Câu 6. (2,0 điểm) Động hóa học hình thức
1. Thực nghiệm cho biết: sau 0,75 giây thì 30ml KOH 1M trung hoà vừa hết 30ml 𝐻2 𝑆𝑂4 0,5𝑀. Hãy xác định
tốc độ của phản ứng đó theo lượng KOH: theo lượng H2SO4. Kết quả thu được ở môi trường hợp đó có hợp lí
không? Tại sao?
2. Hãy đưa ra các biểu thức cần thiết để chứng minh vai trò của hệ số các chất trong phương trình phản ứng
khi xác định tốc độ phản ứng. (dùng phương trình 𝑎𝐴 + 𝑏𝐵 → 𝑑𝐷 + 𝑒𝐸 với giả thiết phương trình đó đủ đơn
giản để dùng trong trường hợp này).
Câu 7. (2,0 điểm) Dung dịch và phản ứng trong dung dịch
1. Trộn 20,00 ml dung dịch H3PO4 0,50 M với 37,50 ml dung dịch Na3PO4 0,40 M, rồi pha loãng bằng nước
cất thành 100,00 ml dung dịch A.
a. Tính pH của dung dịch A.
b. Cần phải thêm bao nhiêu ml dung dịch HCl 0,050 M vào 20,00 ml dung dịch A để thu được dung dịch
có pH =5,00 (metyl đỏ đổi màu).
c. Cần phải thêm bao nhiêu ml dung dịch NaOH 0,10 M vào 25,00 ml dung dịch A để hỗn hợp thu được
có màu đỏ tía của phenolphtalein (pH = 10,00).
Cho biết:
pKa1(𝐻3 PO4 ) =2,15; pKa2(𝐻3PO4 ) =7,21; pKa3(𝐻3 PO4 ) =12,32.
Câu 8. (2,0 điểm) Phản ứng oxi hoá khử. Pin điện và điện phân
+0,68𝑉 +1,77𝑉
1. Cho: 𝑂2 → 𝐻2 𝑂2 → 𝐻2 𝑂
a) So sánh độ bền giữa các dạng oxi hoá - khử
b) Từ dữ kiện trên hãy tính 𝐸𝑂02,𝐻 +/ 𝐻 2𝑂
và E0𝑂2/OH−

c) Thiết lập sự phụ thuộc E - pH của các cặp 𝑂2 /𝐻2 O và O2 /OH−


2. Phản ứng giữa AgNO3 và KCl tạo thành kết tủa AgCl và giải phóng năng lượng.
Ta có thể tạo thành một tế bào quang điện hoá sinh công nhờ phản ứng đó.
a) Viết công thức của tế bào điện hoá theo IUPAC và các nữa phản ứng điện cực tại anốt và catốt.
0 0
b) Tính 𝛥𝐺298 của phản ứng kết tủa AgCl và 𝐸298 của tế bào điện hoá.
Biết 𝑇AgCl = 1,6.10−10
Câu 9. (2,0 điểm) Halogen- Oxi – lưu huỳnh
1. Hòa tan Cl2 và Br2 (tỉ lệ số mol tương ứng là 5:2) trong nước để được 2 lít dung dịch A có khối lượng riêng
d= 1,00675g/ml.

3
Cho vào 2 lít dung dịch A một lượng NaI có khối lượng m gam. Sau khi kết thúc phản ứng, thu được dung
dịch X1. Cô cạn X1 thu được chất rắn B. Tính m để thu được 15,82gam chất rắn B.
2. Đốt cháy hoàn toàn 3 gam một mẫu than có chứa tạp chất S. Khí thu được cho hấp thụ hoàn toàn bởi 0,5 lít
dung dịch NaOH 1,5M được dung dịch A, chứa 2 muối và có xút dư. Cho khí Cl 2 (dư) sục vào dung dịch A,
sau khi phản ứng xong thu được dung dịch B, cho dung dịch B tác dụng với dung dịch BaCl 2 dư thu được m
gam kết tủa, nếu hoà tan lượng kết tủa này vào dung dịch HCl dư còn lại 3,495 gam chất rắn.
a. Tính  khối lượng C; S trong mẫu than.
b. Tính m gam kết tủa?
Câu 10. (2,0 điểm) Đại cương hữu cơ (quan hệ giữa cấu trúc và tính chất)
1. Hãy cho biết sự tương quan lập thể giữa hai hợp chất trong mỗi cặp sau đây, giải thích ngắn gọn.

2. Dự đoán sản phẩm chính khi cho mỗi chất sau đây tác dụng với Br2, FeBr3

3. Giải thích tại sao brom hóa biphenyl xảy ra tại vị trí orto và para hơn là vị trí meta.

……………………….HẾT…………………………

4
TRƯỜNG THPT CHUYÊN ĐỀ THI CHỌN HSG KHU VỰC DUYÊN HẢI BẮC BỘ
CHU VĂN AN – BÌNH ĐỊNH NĂM HỌC 2021 – 2022
Môn: Hóa học – Lớp 10
Đề đề xuất Thời gian làm bài 180 phút (không kể thời gian phát đề)

Câu 1: (2 điểm) Cấu tạo nguyên tử, phân tử, định luật tuần hoàn
1.1. Nitrogen dioxide (NO2) là hợp chất màu nâu đỏ và có tính thuận từ. Khi làm lạnh từ từ, NO2 dần
dần dần mất màu do chuyển hóa thành Dinitrogen tetraoxide (N2O4).
a) Viết 2 kiểu công thức Lewis có thể có của Nitrogen dioxide, biết electron độc thân trên 2 công thức lần
lượt đặt trên nguyên tử N và nguyên tử O.
b) Từ 2 công thức trên viết 3 công thức khả dĩ của N2O4.
c) 3 dạng N2O4 này tồn tại ở trạng thái lỏng, dạng chủ yếu ứng với phân tử có chứa liên kết nito-nito (Dạng
phân tử X). Phân tử Hydrazine (H2N–NH2) có cấu trúc tương tự. Biết năng lượng liên kết N-N trong 2 phân
tử là 60 và 150 kJ.mol-1. Hãy gán giá trị này cho 2 phân tử và so sánh độ dài liên kết N-N của chúng.
1.2. Nguyên tố R thuộc chu kì 3. Nguyên tử A có các giá trị năng lượng ion hóa như sau (kJ/mol)
I1 I2 I3 I4 I5 I6 I7 I8
1000 2251 3361 4564 7013 8495 27106 31669
a) Xác định R.
b) Một số florua của R gồm: RF4, RF6. Cho biết trạng thái lai hóa của R trong các hợp chất trên và cấu trúc
hình học của các hợp chất đó.
Câu 2. (2 điểm): Tinh thể
Bạc có bán kính nguyên tử R = 144 pm, kết tinh dạng lập phương tâm mặt. Tùy theo kích thước mà các
nguyên tử lạ A có thể phân bố vào mạng tinh thể bạc tạo ra các dung dịch rắn có tên gọi khác nhau: dung dịch
rắn xen kẽ (bằng cách chiếm các lỗ xen kẽ), hoặc dung dịch rắn thay thế (thay thế các nguyên tử bạc).
a) Tính khối lượng riêng của bạc nguyên chất (kg.m-3).
b) Vàng có bán kính R’ bằng 147 pm, được phân bố vào tinh thể bạc tạo dung dịch rắn X thay thế có thể
biểu diễn bằng phương trình:
xAu + Ag ⎯⎯⎯⎯→ AuxAg1-x + xAg
Tính độ dài ô mạng cơ sở aX (pm) theo x (với x là phần mol của Au).
c) Một hợp kim bạc vàng có tỉ lệ khối lượng của Au (kí hiệu w) bằng 10%. Xác định giá trị của x và khối
lượng riêng của hợp kim.
Cho biết : MAu = 197 g.mol-1; MAg = 108 g.mol-1
Câu 3: (2 điểm): Phản ứng hạt nhân
Đồng vị 131
53 I dùng trong y học thường được điều chế bằng cách bắn phá bia chứa Te bằng nơtron
130
52

trong lò phản ứng hạt nhân. Trong phương pháp này, trước tiên 130
52Te nhận 1 nơtron chuyển hóa thành 131
Te ,
52

rồi đồng vị này phân rã  tạo thành


- 131
53 I.
a) Viết phương trình các phản ứng hạt nhân xảy ra khi điều chế 131
53 I.
b) Trong thời gian 3 giờ, 1 mL dung dịch 131
53 I ban đầu phát ra 1,08.1014 hạt -.
- Tính nồng độ ban đầu của 131
53 I trong dung dịch theo đơn vị mol/L.
- Sau bao nhiêu ngày, hoạt độ phóng xạ riêng của dung dịch 131
53 I chỉ còn 103 Bq/mL?
131
Biết chu kì bán rã của 53 I là 8,02 ngày.
Câu 4: (2 điểm): Nhiệt hóa học

5
4.1. Người ta đưa 3,6 gam một hiđrocacbon X (khí) cùng một lượng dư oxi vào một bom nhiệt kế ban
đầu chứa 600 gam nước tại 200C. Sau phản ứng xong nhiệt độ của nhiệt lượng kế lên tới 280C, thấy có 11 gam
CO2 (khí) và 5,4 gam H2O (lỏng) tạo thành.
Cho biết: - Nhiệt sinh chuẩn của CO2 (khí) là -393,51 kJ/mol; của H2O (lỏng) là 285,83 kJ/mol.
- Nhiệt dung riêng của nước là 4,184 J/g.K
- Biến thiên nội năng của phản ứng trên là U 0 = −2070,00 kJ/mol
a) Xác định công thức phân tử của X và tính nhiệt dung riêng của nhiệt lượng kế.
b) Xác định nhiệt sinh chuẩn của X.
4.2. Hoá lỏng 1mol khí oxi ở -1830C bằng cách nén ở áp suất 1 atm. Oxi sau khi đã hoá lỏng được làm
lạnh ở áp suất không đổi đến nhiệt độ nóng chảy là -2180C, sau đó được hoá rắn thuận nghịch và chất rắn sau
đó được làm lạnh đến -2630C. Hãy tính H (hệ) và S (hệ) của toàn bộ quá trình trên.
Cho: Cp (l) = 54 J.K−1.mol−1; Cp (r) = 41 J.K−1.mol−1; H (hoá hơi) = 6,82 kJ.mol−1; H (nóng chảy) = 0,42
kJ.mol−1.

Câu 5: (2 điểm): Cân bằng hóa học trong pha khí

Xét cân bằng: N2O4(k) ⎯⎯⎯ → 2NO2(k)



a) Một lượng khí N2O4 được đặt trong một xilanh ở nhiệt độ T1 = 250C. Sau khi cân bằng được thiết lập,
áp suất chung của hệ là 1,5 atm và có 16% số mol của N2O4 bị phân hủy thành NO2. Tính áp suất ban đầu của
N2O4 và Kp của phản ứng phân hủy ở 250C.
b) Nếu nhiệt độ vẫn giữ ở 200C nhưng tăng dần thể tích của xilanh cho tới khi áp suất chung của hệ bằng
1 atm. Tính áp suất cân bằng của NO2 và N2O4 trong điều kiện này.
c) Tính phần trăm số mol của N2O4 đã bị phân hủy ở trạng thái cân bằng mới (áp suất chung là 1atm,
0
25 C).
d) Cho sinh nhiệt tiêu chuẩn ở 298K của N2O4(k) bằng 9,2 kJ/mol và của NO2(k) bằng 33,2 kJ/mol. Tính
S phản ứng và nhiệt độ T2 để phản ứng phân hủy N2O4 có hằng số cân bằng Kp bằng 1. Coi H0pư và S0pư
0

không phụ thuộc nhiệt độ.


Câu 6: (2 điểm): Động hóa học hình thức
Ở nhiệt độ thích hợp, SO2Cl2 tự phân hủy theo phản ứng: SO2Cl2(k) → SO2(k) + Cl2(k). Động học của
phản ứng này được nghiên cứu bằng cách cho một lượng SO2Cl2 vào một bình kín (ban đầu không chứa chất
nào khác) và đo áp suất của hệ theo thời gian.
Kết quả thu được ở 270oC như sau:
t (phút) 0 50 100 150 200 250
P (mmHg) 310,0 334,0 356,1 376,5 395,4 412,8
a) Chứng minh rằng ở nhiệt độ này phản ứng tuân theo quy luật động học bậc 1.
b) Tính hằng số tốc độ phản ứng và thời gian bán phản ứng ở 270oC.
c) Tính áp suất của hệ tại thời điểm t = 275 phút.
d) Ở 2800C, kết quả nghiên cứu phản ứng trên như sau:
t (phút) 0 185 370
P (mmHg) 400 600 700
Tính thời gian bán phản ứng ở 280 C và chỉ ra rằng bậc của phản ứng không thay đổi trong khoảng
o

nhiệt độ nghiên cứu từ 2700C -2800C.


Câu 7: (2 điểm): Dung dịch và phản ứng trong dung dịch
Trộn 10,00 ml dung dịch CH3COOH 0,02M với 10,00 ml dung dịch H3PO4 thu được dung dịch A có
pH = 1,5.

6
a) Tính nồng độ mol/l của H3PO4 trong dung dịch trước khi trộn.
b) Tính độ điện li của CH3COOH trong dung dịch A.
c) Thêm từ từ Na2CO3 rắn vào dung dịch A cho đến pH = 4,0 thu được dung dịch B. Tính số gam Na2CO3
đã dùng ( cho rằng khí CO2 tạo thành thoát ra hết).
Cho biết: H3PO4 có pK a1 = 2,15; pK a2 = 7,21; pK a3 = 12,32
CH3COOH có pKa = 4,76; H2CO3 có pK a1 = 6,35; pK a2 = 10,33 .

Câu 8: (2 điểm): Phản ứng oxi hoá khử. Pin điện và điện phân
8.1. Ở 25°C, Pin điện hóa sau có giá trị suất điện động là 0,824 (V).
Pt | I‒ 0,1 (M); I3‒ 0,02 (M) || MnO4‒ 0,05 (M); Mn2+ 0,01 (M); HSO4‒ x (M) | Pt
Tính x.
Cho E°(MnO4‒/Mn2+) = 1,51 (V); E°(I3‒/3I‒) = 0,5355 (V); Ka(HSO4‒) = 10‒2;
RT
×ln = 0,0592×lg.
F
8.2. Dung dịch X gồm K2Cr2O7 0,010 M; KMnO4 0,010 M; Fe2(SO4)3 0,0050 M và H2SO4 (pH của dung dịch
bằng 0). Thêm dung dịch KI vào dung dịch X cho đến nồng độ của KI là 0,50 M, được dung dịch Y (coi thể
tích không thay đổi khi thêm KI vào dung dịch X).
a) Hãy mô tả các quá trình xảy ra và cho biết thành phần của dung dịch Y.
b) Tính thế của điện cực platin nhúng trong dung dịch Y.
c) Cho biết khả năng phản ứng của Cu2+ với I- (dư) ở điều kiện tiêu chuẩn. Giải thích?
0 0 0 0
Cho: E 2− = 1,330 V; E − = 1,510 V; E = 0,771 V; E − − = 0,5355 V
Cr2 O 7 /Cr 3+ MnO 4 /Mn
2+
Fe
3+
/Fe
2+
I3 /I

0
E 2+ + = 0,153 V; pKs(CuI) = 12.
Cu /Cu

Câu 9: (2 điểm): Halogen, Oxi – lưu huỳnh


Nguyên tố X có trong thành phần của các acid A1, A2 và A3. Phần trăm khối lượng các nguyên tố trong
A1, A2 và A3 như sau:
Chất %O %H %X
A1 46,72 1,46 51,82
A2 56,81 1,18 42,01
A3 63,68 1,00 35,32
a) Xác định X, A1, A2 và A3. Sắp xếp theo chiều tăng dần tính acid của A1, A2, A3 và giải thích.
b) Đơn chất của X phản ứng với dung dịch KOH lạnh, được dung dịch chứa X1. Đun nóng dung dịch này
được dung dịch chứa X2. Hợp chất X1 phản ứng với KI ngay trong môi trường trung tính trong khi X2 chỉ
phản ứng với KI trong môi trường acid. Oxi hóa X2 trong điều kiện thích hợp được X3. Hợp chất X2 phản
ứng với SO2 hoặc oxalic acid trong môi trường sulfuric acid đều tạo ra chất khí X4 màu vàng lục, tan nhiều
trong nước, chứa 52,59% nguyên tố X về khối lượng. Hấp thụ X4 vào dung dịch KOH, được dung dịch chỉ
chứa X2 và X5. Hợp chất X4 cũng được tạo thành khi cho X5 phản ứng với dung dịch HCl. Biết X1, X2, X3
và X5 là các muối tan trong nước.
Biện luận và xác định X1, X2, X3, X4, X5. Viết phương trình hóa học của các phản ứng liên quan tới
X4 ở trên.
Câu 10: (2 điểm): Đại cương hữu cơ
10.1. Nghiên cứu cấu trúc hợp chất (2R,3S)-2,3-dichloro-1,4-dioxane dưới đây bằng phương pháp nhiễu xạ
tia X, người ta thấy độ dài của liên kết C-Cl trục (1,819 Å) lớn hơn của liên kết C-Cl biên (1,781 Å). Đồng

7
thời, độ dài của liên kết C-O của nguyên tử C mang liên kết C-Cl trục (1,394 Å) lại ngắn hơn liên kết C-O của
nguyên tử C mang liên kết C-Cl biên (1,425 Å). Giải thích các giá trị thực nghiệm này.

10.2.
a) Hãy cho biết hợp chất nào dễ tham gia phản ứng SN1 nhất trong số ba hợp chất dưới đây?

.
b) Hãy biểu diễn cấu trúc của hợp chất 2-đeoxy-D-ribozơ [Tên hệ thống: (3S,4R)-3,4,5 trihyđroxipentanal]
dưới dạng công thức chiếu Fischer và công thức phối cảnh.
10.3. Hãy giải thích sự biến đổi lực base của các hợp chất dưới đây:

pKa 10,58 7,79 5,06

---------------------------Hết ------------------------------

ĐỀ THI DHBB MÔN HÓA HỌC – LỚP 10 - NĂM 2022


TRƯỜNG THPT CHU VĂN AN
Câu 1. (2,0 điểm) Cấu tạo nguyên tử, phân tử, định luật tuần hoàn
1. Trong bảng dưới đây có ghi các giá trị năng lượng ion hóa liên tiếp In (n = 1, 2, ..., 6) theo kJ.mol-1 của hai
nguyên tố X và Y:
I1 I2 I3 I4 I5 I6
X 590 1146 4941 6485 8142 10519
Y 1086 2352 4619 6221 37820 47260
Electron cuối cùng của X và Y ở trạng thái cơ bản đều có tổng giá tri bốn số lượng tử là 3,5.
Tìm tên nguyên tố và viết cấu hình electron của X và Y.
2. Sử dụng mô hình VSEPR, hãy dự đoán và vẽ cấu trúc hình học của các phân tử và ion sau: ClOF2+, ClOF3,
ClOF4-. Hãy cho biết trạng thái lai hóa của nguyên tử trung tâm trong các phân tử và ion trên.
Câu 2. (2,0 điểm) Tinh thể
Sắt có 4 dạng thù hình tùy theo điều kiện nhiệt độ như sau:
Nhiệt độ Dạng thù hình Cấu tạo mạng tinh thể Độ dài cạnh a (Ao)
8
to ≤ 770oC Feα Lập phương tâm khối 2,86
770 < to < 910oC Feβ Lập phương tâm khối 2,90
910 ≤ to ≤ 1390oC Feγ Lập phương tâm diện 3,56
to > 1390oC Feδ Lập phương tâm khối 2,93
1. Một viên bi sắt có khối lượng 3,1977 gam ở nhiệt độ thường, được nung nóng đến 1000oC, thể tích viên bi
sắt đó tăng hay giảm bao nhiêu % so với ban đầu?
2. Cho một viên bi sắt có khối lượng 3,1977 gam ở nhiệt độ thường vào 50,00 gam dung dịch HCl 15,00%
đến khi bán kính viên bi sắt còn lại một nửa thì lấy viên bi sắt ra (giả sử quá trình ăn mòn viên bi sắt xảy ra
đồng đều theo tất cả các hướng và viên bi có dạng hình cầu). Tính nồng độ phần trăm các chất trong dung
dịch sau phản ứng.
Cho Fe = 55,847 đvC; số Avogađro = 6,023.1023.
Câu 3. (2,0 điểm) Phản ứng hạt nhân
Ngày nay, urani tự nhiên gồm vài đồng vị. Các đồng vị có chu kì bán hủy dài là: 238U (chiếm 99,275%) có t1/2
= 4,468.109 năm và 235U (chiếm 0,720%) có t1/2 = 7,038.108 năm.
Ngoài ra còn có 234U chiếm 0,005% urani tự nhiên. Đồng vị này không có khi trái đất hình thành mà được tạo
thành trong quá trình phân rã của một trong các đồng vị trên. Hiện nay lượng 234U không thay đổi nữa.
1. Tính thời điểm (trong quá khứ) mà khối lượng 238U gấp đôi khối lượng 235U.
2. Tính chu kì bán hủy của 234U.
Câu 4. (2,0 điểm) Nhiệt hóa học
1. Biết giá trị nhiệt động của các chất sau ở điều kiện chuẩn (298K):
Fe O2 FeO Fe2O3 Fe3O4
∆Hos (kcal.mol-1) 0 0 -63,7 -169,5 -266,9
So (cal.mol-1.K-1) 6,5 49.0 14,0 20,9 36,2
Tính ∆Go của sự tạo thành các oxit sắt từ các đơn chất ở điều kiện chuẩn. Từ đó cho biết ở điều kiện chuẩn
oxit sắt nào bền nhất?
2. Tính ∆S của quá trình hóa hơi 4 mol H2O (l) ở 25oC, 1 atm.
Biết: ∆Hhh, H2O (l) = 40,656 kJ.mol-1 và nhiệt dung đẳng áp của nước lỏng và hơi nước lần lượt là 75,291 và
33,580 (J.mol-1.K-1).
Câu 5. (2,0 điểm) Cân bằng hoá học trong pha khí
Xét phản ứng tách H2 của etan: C2H6 CH2=CH2 + H2 (1)
Cho các số liệu sau đối với phản ứng trên: ∆Go 900K = 22,39 kJ.mol-1.
H2 Etan Etilen
So 900K(J.mol-1.K-1) 163,0 319,7 291,7
1. Tính KP của phản ứng (1) ở 900K. Tại trạng thái cân bằng của phản ứng (1), áp suất của hệ là 2 atm. Tính
KC, Kx.

9
2. Tính thành phần phần trăm theo thể tích của các chất tại cân bằng của phản ứng (1), biết áp suất của hệ tại
trạng thái cân bằng là 1 atm.
3. Ở 900K, phản ứng CH2=CH2 + H2 C2H6 (2) tỏa nhiệt hay thu nhiệt?
4. Tính KP của phản ứng (1) ở 600K, giả thiết ∆Ho và ∆So không thay đổi theo nhiệt độ.
Câu 6. (2,0 điểm) Động hóa học hình thức
1. Cho phản ứng 2N2O5 4NO2 + O2 ở nhiệt độ T (K), với các kết quả thực nghiệm sau:
Thí nghiệm 1 Thí nghiệm 2 Thí nghiệm 3
Nồng độ N2O5 (mol.l-1) 0,170 0,340 0,680
Tốc độ phân hủy (mol.l-1.s-1) 1,39.10-3 2,78.10-3 5,55.10-3
a. Hãy viết biểu thức tốc độ của phản ứng trên.
b. Biết năng lượng hoạt hóa (EA) của phản ứng trên là 24,74 Kcal.mol-1 và ở 25oC nồng độ N2O5 giảm đi một
nửa sau 341,4 giây. Hãy tính nhiệt độ T.
Câu 7. (2,0 điểm) Dung dịch và phản ứng trong dung dịch
Dung dịch A gồm H3PO4 x (M) và NaHSO4 0,010M có pHA = 2,03.
1. Tính x.
2. Tính nồng độ HCOOH phải thêm vào dung dịch A sao cho độ điện li của H3PO4 giảm 25% (coi thể tích
dung dịch không thay đổi).
Biết: H3PO4 có pKa1 = 2,15; pKa2 = 7,21; pKa3 = 12,32. pKa (HSO4-) = 2; pKa (HCOOH) = 3,75.
Câu 8. (2,0 điểm) Phản ứng oxi hoá khử. Pin điện và điện phân
1. Lập các phương trình hóa học của các phản ứng sau theo phương pháp ion-electron:
a. CrO2- + Br2 + OH- → CrO42- + ...
b. Cu2FeSx + KHSO4 + KNO3 → SO42- + NO + ...
c. C6H5-CH=CH2 + KMnO4 + H2SO4 → C6H5COOH + CO2 + ...
2. Có hai bình điện phân mắc nối tiếp nhau. Bình (1) chứa V lít dung dịch CuCl2 2x (M); bình (2) chứa 2V lít
dung dịch AgNO3 x (M). Thực hiện sự điện phân các dung dịch trên với các điện cực trơ trong thời gian 50
phút, cường độ dòng điện không đổi 1,93A.
Trộn hai dung dịch sau điện phân với nhau, thu được kết tủa và dung dịch B (có chứa 0,08 mol Cl-). Viết các
phương trình hóa học và tính khối lượng mỗi muối trong các dung dịch ban đầu.
Câu 9. (2,0 điểm) Halogen, Oxi – lưu huỳnh
1. Nạp 1 mol SO2 và 0,7 mol O2 vào một bình kín (có chất xúc tác V2O5), nung bình ở nhiệt độ thích hợp để
phản ứng xảy ra. Dẫn từ từ toàn bộ hỗn hợp thu được sau phản ứng vào dung dịch Ba(OH)2 dư để phản ứng
xảy ra hoàn toàn, thu được 223,4 gam kết tủa. Tính hiệu suất của phản ứng ban đầu.
2. Nung nóng hỗn hợp G (gồm bốn muối của natri X, Y, Z, T có cùng a mol mỗi muối) đến 200oC thoát ra khí
E không duy trì sự cháy và hỗn hợp chất rắn M (chứa 4a/3 mol X, 5a/3 mol Z, a mol T) có khối lượng giảm
12,5% so với hỗn hợp G. Nếu tiếp tục nung hỗn hợp M đến 400oC thì chỉ thu được hỗn hợp chất rắn chứa X
và T. Nếu tiếp tục nung đến 600oC thì chỉ còn duy nhất chất X.

10
a. Xác định X, Y, Z, T biết rằng X chỉ gồm hai nguyên tố với phần trăm khối lượng của natri nhỏ hơn phần
trăm khối lượng của nguyên tố còn lại là 21,36%.
b. Xác định phần trăm khối lượng mỗi chất trong hỗn hợp G.
Câu 10. (2,0 điểm) Đại cương hữu cơ (quan hệ giữa cấu trúc và tính chất)
1. Nhiệt độ nóng chảy của cis-but-2-en (-139oC) thấp hơn nhiệt độ nóng chảy của trans-but-2-en (-105oC)
nhưng nhiệt độ sôi của cis-but-2-en (4oC) lại cao hơn nhiệt độ sôi của trans-but-2-en (1oC). Hãy giải thích.
2. So sánh lực axit của các chất sau và giải thích:
CH3-CHBr-CH2-COOH; CH3-CH2-CHBr-COOH; CH3-CH2-CH2-COOH; CH3-CH2-CHCl-COOH; CH3-
CHBr-CHCl-COOH.
-----------------------HẾT---------------------
SỞ GIÁO DỤC VÀ ĐÀO TẠO BẮC GIANG ĐÁP ÁN ĐỀ THI ĐỀ XUẤT DUYÊN HẢI BẮC BỘ
TRƯỜNG THPT CHUYÊN BẮC GIANG Năm 2022
-----***----- Môn: HÓA HỌC – LỚP 10
Thời gian làm bài: 180 phút

Câu 1. (2,0 điểm) Cấu tạo nguyên tử, phân tử, định luật tuần hoàn
1. Một vạch phổ phát xạ cho Be3+ có bước sóng 253,4nm ứng với một chuyển dịch electron từ mức n=5 về
mức nt thấp hơn. Hãy tính nt
2. Hình bên dưới là phổ phát xạ cho ion một electron trong pha khí. Các vạch phổ tương ứng với bước chuyển
electron từ các trạng thái kích thích về trạng thái có n=3.

a. Hãy cho biết chuyển dịch electron tương ứng với các vạch A và B.
b. Nếu bước sóng tương ứng với vạch B là 142,5nm thì bước sóng tương ứng với vạch A là bao nhiêu?
3. Năng lượng ion hóa cho một electron trong ion một electron Mm+ là 4,72  104 kJ/mol. Xác định nguyên tố
X và giá trị m.

Câu 2. (2,0 điểm) Tinh thể


Trong mạng tinh thể của Beri borua, nguyên tử Bo kết tinh ở mạng lưới lập phương tâm mặt và trong đó tất
cả các hốc tứ diện đã bị chiếm bởi nguyên tử beri. Khoảng cách ngắn nhất giữa 2 nguyên tử Bo là 3,29A0.
1. Vẽ hình biểu diễn sự chiếm đóng của nguyên tử Bo trong một ô mạng cơ sở.

2. Có thể tồn tại bao nhiêu hốc tứ diện, hốc bát diện trong một ô mạng? Từ đó cho biết công thức thực
nghiệm của hợp chất này ( công thức cho biết tỉ lệ nguyên tử của các nguyên tố). Trong một ô mạng
cơ sở có bao nhiêu đơn vị công thức trên?

11
3. Cho biết số phối trí của Be và Bo trong tinh thể này là bao nhiêu?

4. Tính độ dài cạnh a0 của ô mạng cơ sở , độ dài liên kết Be-B và khối lượng riêng của beri borua theo
đơn vị g/cm3. Biết Be: 10,81 ; Bo 9,01

Câu 3. (2,0 điểm) Phản ứng hạt nhân


134
Cs và Cs là sản phẩm phân hạch của nhiên liệu urani trong lò phản ứng hạt nhân. Cả hai đồng vị này đều
137

phân rã β- với thời gian bán hủy là t1/2(134Cs) = 2,062 năm và t1/2(137Cs) = 30,17 năm.
Trong một mẫu nước thu được sau sự cố của nhà máy điện hạt nhân người ta phát hiện được các đồng vị nói
trên của Cs với các hoạt độ phóng xạ: A (137Cs) = 0,128 mCi và A (134Cs) = 0,64.µCi. Sau bao nhiêu năm thì
hoạt độ phóng xạ tổng cộng của 2 đồng vị này trong mẫu nước đã cho chỉ còn bằng 8,0 µCi? Tính tỉ số khối
lượng của 134Cs và 137Cs tại thời điểm đó. Giả thiết rằng thiết bị đo chỉ đo được các hoạt độ phóng xạ β- lớn
hơn 0,1 Bq. Cho 1Ci = 3,7.1010 Bq.

Câu 4. (2,0 điểm) Nhiệt hóa học


Cho 150 gam CO2 ở 273,15 K và 1,01325.105 Pa. Xác định nhiệt, công, biến thiên nội năng, biến thiên
entanpi trong các quá trình sau đây được tiến hành thuận nghịch nhiệt động:
1. Dãn đẳng nhiệt đến thể tích 300 lít.
2. Dãn đẳng áp đến thể tích 200 lít.
3. Đun nóng đẳng tích tới áp suất gấp ba lần áp suất lúc ban đầu.
Chấp nhận rằng CO2 là khí lí tưởng và nhiệt dung đẳng áp của nó không đổi trong điều kiện khảo sát và
bằng 37,1 J.mol-1.K-1.

Câu 5. (2,0 điểm) Cân bằng hoá học trong pha khí
Bơm khí SO3 vào bình rồi nâng nhiệt độ lên 900K. Ở trạng thái cân bằng, áp suất tổng là 1,306 atm và tỉ lệ
PSO3
= 2,58.
PSO2
1
a. Tính Kp của cân bằng SO3(k) SO2(k) + O2(k) .
2
b. Khi có xúc tác V2O5, giả thuyết có cân bằng V2O5(r) + SO2(k) V2O4(r) + SO3(k) , người ta đo được bằng thực
PSO3
nghiệm giá trị lg ở hai nhiệt độ 900K và 830K tương ứng là ‒1,7 và ‒1,82. Tính G of tại 25oC, giả
PSO2

thuyết H of và Sof không phụ thuộc vào nhiệt độ. Biết ln = 2,3  lg
c. Tính PO2 tương ứng với sự phân hủy V2O5(r) thành V2O4(r) ở 900K.

Câu 6. (2,0 điểm) Động hóa học hình thức

12
Tốc độ đầu của sự phân hủy ozon thành oxi được khảo sát dưới các điều kiện khác nhau bằng cách đo sự thay
đổi áp suất khi phản ứng diễn ra.
2 O3 (k) → 3 O2(k)
Dữ kiện dưới đây cho ở 90oC, sự có mặt lượng nhỏ O3 so với O2 (O2 coi như không đổi):
p(O3), mmHg ∆P/∆t, mmHg.s-1
7,9 1,21.10-3
17,7 5,8.10-3

1. Nếu áp suất của hệ thay đổi 1,21.10-3 mmHg.s-1 ở 90oC thì tốc độ biến mất của O3 là bao nhiêu mol.L-1.s-1?
2. Bậc riêng phần của O3 trong điều kiện này?
Trong một điều kiện khác, áp suất đầu của O3 được giữ cố định, tốc độ đầu đo được là hàm của áp suất O2 ở
90oC và ở 100oC.
p(O2), mmHg ∆P/∆t, mmHg.s-1, ở 90oC ∆P/∆t, mmHg.s-1, ở 100oC
200 3,30.10-3 7,40.10-3
400 1,45.10-3 3,64.10-3
3. Xác định bậc của O2?
4. Tính năng lượng hoạt hóa của phản ứng?
Câu 7. (2,0 điểm) Dung dịch và phản ứng trong dung dịch
1. Thêm NaOH 0,1M vào dung dịch hỗn hợp các axit H3PO4 0,02M, CH3COOH 0,02M, H3BO3 0,02M (dung
dịch A).
a. Tính pH của dung dịch thu được khi thêm 30ml dung dịch NaOH 0,1M vào 100ml dung dịch A.
b. Thêm V ml dung dịch NaOH 0,1M vào 100 ml dung dịch A ở trên thì thu được dung dịch B có pH = 8,5. Xác
định V?
Cho: H3PO4 có pKa1 = 2,12; pKa2 = 7,21; pKa3 = 12,36. CH3COOH có pKa = 4,75; H3BO3 có pKa = 9,25.
Câu 8. (2,0 điểm) Phản ứng oxi hóa khử, pin điện và điện phân

Cho sơ đồ pin điện hoá tại 25oC :


(-)Ag, AgBr/KBr (1M) || Fe3+ (0,05M), Fe2+ (0,1M)/Pt(+)
a) Viết sơ đồ phản ứng xảy ra trong pin và chiều chuyển dịch điện tích khi pin hoạt động.
b) Tính E pin.
c) Tính nồng độ các ion trong mỗi điện cực khi pin phóng điện hoàn toàn.
o o
Cho: EAg+
/Ag = 0,799V ; EFe3+/Fe2+ = 0,771 V
Ks, AgBr = 10-13 . ThÓ tÝch mçi ®iÖn cùc lµ 100ml
Câu 9. (2,0 điểm) Halogen, Oxi – lưu huỳnh
1. Cho các phản ứng:
(1): A(k) → B(k)
(2): X(r) + A(k) + H2O(l) → H2SO4(dd)
(3): X(r) + B(k) → Y(k)
(4): Y(k) + A(k) + H2O(l) → H2SO4(dd)
(5): Z(k) + X(r) → T(k)
(6): Pb(NO3)2 + T(k) → D(r)  + E(dd)

13
Hoàn thành các phản ứng trên bằng cách tìm các chất vô cơ phù hợp để gán cho mỗi chữ cái in hoa, cân bằng
phản ứng thu được.
2. Giải thích các hiện tượng sau: SnS2 tan trong (NH4)2S; SnS không tan trong dung dịch (NH4)2S nhưng tan
trong dung dịch (NH4)2S2.
Câu 10. (2,0 điểm) Đại cương hữu cơ (quan hệ giữa cấu trúc và tính chất)
1. Gọi tên theo danh pháp IUPAC các chất hữu cơ sau:

2.
a) Quy kết các giá trị nhiệt độ sôi (oC) sau: 116, 194, 201 phù hợp cho 3 đồng phân vị trí của nitrophenol.
Giải thích ngắn gọn.
b) So sánh và giải thích ngắn gọn tính axit của H trong các phân tử sau:
H

H
A B C

3. Xác định cấu dạng bền của các hợp chất A, B trong các môi trường: a) metanol; b) octan.
Cl

Me HO OH
O
A B

-----Hết-----
ĐỀ GIỚI THIỆU CỦA BẮC NINH
KÌ THI CHỌN HỌC SINH GIỎI
THPT CHUYÊN – DUYÊN HẢI BẮC BỘ NĂM 2022
MÔN: HOÁ HỌC - LỚP 10
Thời gian làm bài: 180 phút

Câu 1 (2 điểm): CTNT – PT – ĐLTH


1/ Sử dụng mô hình về sự đẩy nhau của các cặp electron hóa trị (mô hình VSEPR), dự đoán dạng hình học
của các ion và phân tử sau: BeH2, BCl3, NF3, SiF62-, NO2+, I3-.
2/ So sánh và giải thích khả năng tạo thành liên kết π của C và Si.
3/ Trong quá trình trao đổi chất ở cơ thể người, oxi phân tử có thể chuyển thành anion O2-. Anion này là chất
oxi hóa mạnh, có khả năng phá hủy tế bào. Tuy nhiên, một số loại enzym trong cơ thể người có tác dụng xúc
tác để chuyển O2- thành các chất không độc hại. NO được biết là một chất khí độc, khi vào cơ thể người, nó
dễ dàng kết hợp với O2- để tạo thành anion X-. Anion này cũng là một tác nhân oxi hóa mạnh, có khả năng phá
hủy protein, ADN và lipit, gây các bệnh về tim, bệnh Alzheimer, bệnh đa xơ cứng, … Vẽ giản đồ obitan phân
tử của O2- và của NO. Dựa trên giản đồ obital phân tử đã vẽ, hãy lập luận về sự hình thành X- từ O2- và NO.

Câu 2 (2,0 điểm): Tinh thể

14
A là hợp chất tạo nên từ 2 nguyên tố X và Y, tỉ lệ bán kính ion X và Y
trong tinh thể là rY : rx = 1,772. Tế bào tinh thể A có độ đặc khít là 68.27%,
được mô tả ở hình bên, dạng hình hộp chữ nhật với a = b = 4.59 Å. Khối
lượng riêng của A là 4.32 g/cm3. A được sử dụng như chất phụ gia cho kem
chống nắng, đồng thời A có vai trò to lớn trong nền công nghiệp luyện kim,
đặc biệt trong ngành hàng không
1. Xác định bán kính ion X, Y trong A. Xác định công thức chất A.
2. X có thể thu được từ quặng chứa A theo sơ đồ sau:
A → XCl4 → X(thô) → XI4 → X (tinh).
a) Viết các phản ứng trong sơ đồ với đầy đủ điều kiện.
b) Bước cuối cùng trong sơ đồ điều chế là quan trọng nhất: XI4 ⇌X + 2I2 (4)
Cân bằng được thiết lập ở 13000C, hằng số cân bằng KC = 0,86 M.
Trong một bình thể tích 10 L chứa 20 mol tinh thể XI4. Bơm không khí và hơi nước nóng vào bình cho đến
khi nhiệt độ đạt 13000C. Tính khối lượng kim loại X thu được và độ chuyển hoá của phản ứng khi hệ đạt cân
bằng.

Câu 3 (2 điểm): Phản ứng hạt nhân


Có 3 chuỗi phóng xạ tự nhiên và 1 chuỗi phóng xạ nhân tạo. Sau các chuyển hóa phóng xạ alpha và
beta, các chuỗi kết thúc với sự tạo thành các đồng vị bền. Giản đồ dưới đây biểu diễn một trong các chuỗi đó.

Biết rằng trong chuỗi này, X4 = X1 (X là loại phân rã). Phân rã phóng xạ tuân theo quy luật động học phản
ứng bậc nhất: dN/dt = -λN, trong đó λ là hằng số phân rã, N là số hạt nhân phóng xạ ở thời điểm t.
1/ Xác định chu kì bán rã T1/2 của nguyên tố C (theo năm), biết rằng trung bình thì mỗi giây có 1 trong số
7.24∙1012 nguyên tử C bị phân rã.
2/ Xác định nguyên tố C, sau đó là D, E, F - sử dụng các dữ kiện bổ sung sau:
+ sau 500 năm, từ 1 gam C xảy ra phân rã α thì có 9.333∙10-6 mol helium được tạo thành;
+ khối lượng nguyên tử của C lớn gấp 2.533 lần trị số điện tích hạt nhân.
3/ Trong tự nhiên, chỉ có 7 khoáng chất của nguyên tố I được biết đến. Hai trong số chúng là IAsSx và
I4Hg3SbxAs8S20, có hàm lượng I lần lượt là 60 % và 28.6 %.
Xác định các nguyên tố I và J, biết khối lượng nguyên tử của I lớn gấp 2.580 lần điện tích hạt nhân của nó.
4/ Một mẫu phóng xạ nặng 3 gam chứa các đồng vị A1 (35 % về khối lượng, T1/2 = 2.1 ngày) và A2 (65 % về
khối lượng, T1/2 = 4.4 ngày). Số khối của A1 lớn hơn 1 amu so với số khối của A, còn số khối của A2 ít hơn
3 amu so với A. Xác định đồng vị A - "tổ tiên" của chuỗi phóng xạ này, biết rằng sau 12 giờ kể từ khi điều
chế thì độ phóng xạ của mẫu là 4.61∙105 Curie (1 Curie = 3.7∙1010 phóng xạ/giây). Độ phóng xạ của mẫu chứa
nhiều đồng vị được tính theo tổng độ phóng xạ thành phần. Khối lượng nguyên tử của A lớn gấp 2.548 lần trị
số điện tích hạt nhân.
Xác định các hạt nhân B, G, H, K.

Câu 4 (2 điểm): Nhiệt hóa học


1/ Một dung dịch gồm 2,895 g axit cacboxylic yếu X (X no, đơn chức) trong 500 g nước có nhiệt độ đông đặc
là −0,146oC. Biết rằng nếu hòa tan 0,957 g saccarozơ vào 100 g nước thì dung dịch này có nhiệt độ đông đặc
là −0,052oC. Hãy xác định axit, Ka và độ điện ly  của axit trong dung dịch trên.
2/ Cho bảng số liệu sau:

Chất H3PO4(dd) H2PO −4 (dd) HPO 24 − (dd) PO 34− (dd) H+(dd) OH-(dd) H2O(l)
H 0f (kJ.mol-1) -1288 -1296 -1292 -1277 0 -230 -286
S0(J,K-1.mol-1) 158 90 -33 -220 0 -11 70

a/ Hãy tính H 0 ; G0 của phản ứng trung hòa từng nấc axit ortho – photphoric bằng kiềm mạnh:

15
OH- + H3PO4 → H2PO −4 + H2O (1)
-
OH + H2PO −
4 → HPO 2−
4 + H2O (2)
OH- + HPO 24 − → PO 34− + H2O (3)
b/ Xác định các hằng số phân ly của axit ortho – photphoric ở điều kiện chuẩn
c/ Cho các dung dịch axit ortho – photphoric và kiềm NaOH đều có nồng độ 0,1M. Hãy xác định thể tích của
mỗi dung dịch để khi trộn chúng với nhau thu được dung dịch có thể tích 25 mL và tỏa ra 90 jun nhiệt.

Câu 5 (2 điểm): Cân bằng hóa học trong pha khí


Ở điều kiện thường, Selen là chất rắn, phân tử gồm 8 nguyên tử selen. Selen bay hơi, tạo ra pha khí
gồm các dạng Sen cân bằng nhau (n = 2 → 8). Biết sinh nhiệt của Se8(k) là  f H Se0 8( k ) = 40,5 kcal.mol −1 . Biết
hiệu ứng nhiệt  r H 0 (kcal.mol-1) của các quá trình:
Phản ứng 3Se2(k) → Se6(k) 2Se4(k) → Se8(k) 2Se2(k) → Se4(k) Se6(k) → 2Se3(k)
r H 0 -71,4 -35,5 -31,7 53,4
1/ Xác định sinh nhiệt của Se6(k) và Se3(k) theo đơn vị kcal.mol-1. So sánh hai giá trị và giải thích.
2/ Năng lượng trung bình mỗi liên kết trong phân tử Se6(k) là 49,4 kcal.mol-1. Xác định năng lượng liên kết
trong phân tử Se2(k)
3/ Trong một thí nghiệm điều chế hơi selen, người ta nung selen thì thu được hỗn hợp các phân tử với áp suất
tương ứng như sau:
Phân tử Se8(k) Se7(k) Se6(k) Se5(k) Se4(k) Se3(k) Se2(k)
P(kPa) 12 10 9,8 8,7 6,1 2 1,5
Xác định số nguyên tử trung bình n trong phân tử khí Sen
4/ Giá trị n sẽ thay đổi như thế nào nếu:
i/ Tăng áp suất
ii/ Tăng nhiệt độ

Câu 6 (2 điểm): Động hóa học hình thức


Các este bị thuỷ phân trong môi trường nước và quá trình này được xúc tác bởi cả acid và base.
1/ Dung dịch ban đầu được điều chế bằng cách trộn khoảng 20 mL este metyl axetat (khối lượng riêng 𝜌este =
0.933 kg/L), 980 mL nước và một lượng xúc tác axit sunfuric (pH ~ 2, được duy trì không đổi, bỏ qua thể tích
axit). Sự phụ thuộc của nồng độ metyl axetat trong nước theo thời gian cho trong bảng sau:

Thời gian, phút 10 150 400 720 1010


[MeCOOMe], M 0.269 0.246 0.211 0.173 0.144

Hãy xác định bậc phản ứng theo este và tính hằng số tốc độ biểu kiến của phản ứng này.
2/ Khi thuỷ phân metyl axetat có mặt base (dư 50%) thì thu được các dữ kiện sau:

Thời gian (phút) 0 0,5 1 1,75


[MeCOOMe] 0,500 0,338 0,246 0,165
r (M.s-1) 7,39.10-3 3,91.10-3 2,40.10-3 1,35.10-3

Hãy xác định bậc phản ứng của este trong trường hợp này và tính hằng số tốc độ (với thứ nguyên).

Câu 7 (2 điểm): Dung dịch và phản ứng trong dung dịch


Để tạo vị chua cho nước coca – cola, người ta thường thêm H3PO4 với hàm lượng photpho là 160 mg
trong một lít nước này. Ngoài ra, tổng lượng CO2 được nén vào 1 chai chứa 330,0 mL nước coca – cola là
1,10 gam.
1. Giả thiết toàn bộ CO2 tan trong nước coca – cola. Tính độ chua (pH) của nước coca – cola trong chai.
2. Sau khi mở nắp chai coca – cola rồi để cân bằng trong không khí thì pH của nước coca – cola thay đổi như
thế nào? Giải thích.

16
3. Tính thể tích (theo mL) dung dịch NaOH 5,00.10–3 M cần cho vào 10,0 mL nước coca – cola ở ý (2) để thu
được dung dịch có pH = 8,00. Bỏ qua ảnh hưởng của CO2 trong không khí đến thí nghiệm.
4. Men răng có thành phần chủ yếu là hydroxyapatit Ca10(PO4)6(OH)2 (M = 1004 gam.mol–1). Men răng được
duy trì bởi trạng thái hoà tan – lắng đọng của Ca10(PO4)6(OH)2 vì trong nước bọt có chứa canxi và photphat.
Tuy nhiên, độ tan của hydroxyapatit ảnh hưởng rất nhiều bởi sự thay đổi độ axit của dung dịch mà nó tiếp
xúc. Khi tiêu thụ thực phẩm và đồ uống có tính axit có thể gây ra hiện tượng mòn men răng.
Bình thường, nước bọt chứa khoảng 1,0 mM ion canxi và 3,0 mM photpho (ở các dạng của photphat) và
pH của nước bọt khoảng 7,0 (được quyết định bởi các dạng tồn tại của photphat).
a. Xác định nồng độ của các dạng tồn tại chính của photphat trong nước bọt.
b. Xác định tích số tan Ksp của Ca10(PO4)6(OH)2 dựa vào cân bằng hòa tan – lắng đọng của men răng trong
nước bọt ở trạng thái bình thường.
c. Một cậu bé thích uống coca – cola. Hãy cho biết, khi cậu bé ngậm 30,0 mL coca - cola và 10,0 mL nước
bọt có sẵn trong miệng (ở trạng thái bình thường) thì men răng có bị tan ra hay không? Giải thích.
Cho biết: Ở 298 K, H3PO4 có: pKa1 = 2,15; pKa2 = 7,21; pKa3 = 12,32;
(CO2 + H2O) có pKa1 = 6,35; pKa2 = 10,33;
CO2(dd) ⇌ CO2(k) KH = 30,2 atm.M-1;
Ca2+ + H2O ⇌ CaOH+ + H+ *β = 10–12,60.
Hàm lượng CO2 trung bình trong không khí là 0,0385% về số mol; áp suất khí quyển là 1,0 atm; các
thành phần khác trong nước coca – cola không ảnh hưởng đến kết quả tính toán.

Câu 8 (2 điểm): Phản ứng oxi hóa khử. Pin điện và điện phân
Pin điện hoá sau đây dựa trên phản ứng ở pha rắn và hoạt động thuận nghịch ở 1000K dưới dòng khí
O2. Các ion F- khuếch tán thông qua CaF2 (r) ở 1000 K.
(-) MgF2 (r), MgO (r) | CaF2 (r)| MgF2 (r) , MgAl2O4(r), Al2O3(r) (+)
Các nửa phản ứng:
Ở điện cực âm: MgO(r) + 2 F- → MgF2 (r) + ½ O2 (k) +2e
Ở điện cực dương: MgF2 (r) + Al2O3(r) +½ O2 (k) +2e → MgAl2O4(r) + 2F-
1/ Viết phương trình phản ứng tổng cộng khi pin hoạt động. Viết phương trình Nernst cho mỗi nửa pin và cho
cả pin. Tính suất điện động của pin ở 1000K. Coi áp suất O2(k) là như nhau ở 2 điện cực. Nồng độ F- là bằng
nhau ở 2 điện cực và được duy trì bởi dòng khuếch tán ion F- thông qua CaF2 (r). Biết rằng, E0 (ở 1000 K) của
phản ứng là 0,1529V
2/ Tính ∆G0 của phản ứng (ở 1000 K).
3/ Sức điện động chuẩn của pin trong khoảng nhiệt độ từ 900 K đến 1250 K là:
E0 (V) = 0,1223 + 3,06. 10-5T. Giả thiết ∆H0, ∆S0 là hằng số, tính các giá trị này.

Câu 9 (2 điểm): Halogen, oxi, lưu huỳnh


1/ Cho 58 mg một oxit kim loại MO2, dung dịch HCl đặc và dung dịch KI được chuẩn bị trong các dụng cụ
thí nghiệm như hình vẽ dưới đây:

Khí CO2 giúp điều chỉnh tốc độ nhỏ giọt của dung dịch HCl đặc trên phễu. Ống nghiệm A được đậy kín
như hình vẽ, sau đó được đun sôi trong khoảng 30 phút. Trong suốt thời gian này, khí CO 2 vẫn được duy trì
để đẩy HCl xuống còn sản phẩm thoát ra trong phản ứng sẽ được chưng cất dần sang bình B và C. Kết thúc
thí nghiệm, toàn bộ dung dịch trong bình C được chuyển sang bình eclen B. Sau đó chuẩn độ dung dịch trong
bình eclen B bằng dung dịch Na2S2O3 0,1 M cho đến khi dung dịch nhạt màu, thêm ít hồ tinh bột vào rồi chuẩn
17
độ đến khi dung dịch mất màu hoàn toàn. Thể tích dung dịch Na2S2O3 đã dùng trong phép chuẩn độ trên là
4,85 mL.
a/ Viết tất cả các phương trình phản ứng xảy ra trong thí nghiệm trên.
b/ Xác định kim loại trong oxit MO2 đã dùng.
2/ Viết phương trình phản ứng khi cho dung dịch chứa Fe2(SO4)3; FeSO4; MgSO4; CuSO4 lần lượt tác dụng
với dung dịch BaS và dung dịch bão hòa H2S.
Biết: E0 của Fe3+ / Fe2+ = 0,771V; S / S2- = - 0,48V; pKa của H2S = 7,02; 12;90
pKs của Fe(OH)3 = 37; FeS = 17,2.

Câu 10(2 điểm): Đại cương hữu cơ (quan hệ giữa cấu trúc và tính chất)
1.Cho hai phân tử naphthalene và phenanthrene.

a. Đối với phân tử naphthalene, Giải thích tại sao độ dài liên kết C2-C3(1.42 Å) lớn hơn độ dài liên kết C1-
C2 (1.36 Å).
b. Chỉ ra và giải thích liên kết có độ dài ngắn nhất trong phân tử phenanthrene.
c. Phản ứng của phenanthrene với dung dịch Br2 tạo hợp chất C14H10Br2. Viết cấu trúc sản phẩm thu được
2. Cho các hợp chất sau:.

Giải thích các vấn đề sau


a. Hợp chất A tồn tại chủ yếu dạng enol B
b. Hợp chất C là hợp chất thơm
c. D làm mất Br- tạo thành cacbocation dễ hơn dẫn xuất E
d. So sánh tính axit của F và G

------------------------------
HỘI CÁC TRƯỜNG CHUYÊN ĐỀ ĐỀ XUẤT
VÙNG DUYÊN HẢI VÀ ĐỒNG BẰNG BẮC BỘ MÔN HÓA HỌC - KHỐI 10
Ngày thi:
Thời gian làm bài: 180 phút

ĐỀ ĐỀ XUẤT

Câu 1 (2,0 điểm) Cấu tạo nguyên tử, phân tử, định luật tuần hoàn
1. Sự phân li của phân tử clo là một quá trình thu nhiệt, ΔH = 243,6 kJ.mol-1. Sự phân li có thể cũng
xảy ra bởi ánh sáng. Ở bước sóng bao nhiêu thì sự phân li xảy ra?
2. Ánh sáng có thể làm xảy ra quá trình phân li khi chiếu vào một hỗn hợp gồm khí clo và hiđro, hiđro
clorua được hình thành. Hỗn hợp được chiếu với đèn thủy ngân UV (λ = 253,6 nm).

18
Đèn có công suất tiêu thụ là 10W. Một lượng 2% năng lượng cung cấp được hấp thụ bởi hỗn hợp khí
(trong một bình 10 L). Trong 2,5 giây chiếu xạ, 65 mmol của HCl được sinh ra. Hiệu suất lượng tử bằng bao
nhiêu?
3. Tính động năng của electron bị tách ra từ quá trình ion hóa ion Li2+ ở trạng thái cơ bản bằng một
photon có tần số 5,00.1016 s-1.
4. Sắp xếp năng lượng ion hóa của các tiểu phân sau đây theo thứ tự từ thấp đến cao O, O2, O2-, O2+.
Giải thích.

Câu 2 (2,0 điểm) Tinh thể

1. Hợp chất Na2O, CdS và ZrI4 tất cả đều có anion kết tinh dạng lập phương tâm diện còn cation chiếm
hốc tứ diện. Tỉ lệ hốc tứ diện bị chiếm trong mỗi trường hợp là bao nhiêu?
2. Cấu trúc của NiO giống như cấu trúc của NaCl. Ion O2- được sắp xếp mạng lưới lập phương tâm
mặt, tất cả các hốc bát diện được chiếm bởi ion Ni2+.
Khối lượng riêng của NiO là 6,67g/cm3.
Nếu xử lí NiO với Li2O và O2 tạo thành tinh thể trắng có thành phần LixNi1-xO là chất bán dẫn tốt.
Phương trình: 1/2x Li2O + (1-x)NiO + 1/4x O2 → LixNi1-xO
Cấu trúc của LixNi1-xO giống như của NiO, tuy nhiên một số nguyên tử Ni được thay thế bởi các
nguyên tử Li và một số ion Ni2+ được oxi hóa để thiết lập sự trung hòa điện tích.
Một chất bán dẫn với khối lượng riêng là 6,21 g/cm3 được tạo ra.
a) Tính x (giả sử: thể tích của ô mạng cơ sở không thay đổi).
b) Tính phần trăm của ion Ni3+ theo tất cả các ion niken trong tinh thể bán dẫn.
Câu 3 (2,0 điểm) Phản ứng hạt nhân

1. Hoàn thành các phương trình phản ứng hạt nhân sau đây:
a) 45
21 Sc + 01n → 1942 K + ? b) Cu + 11H → ?+ 01n
63
29

2. Thời gian bán phản ứng của 47Ca là 4,536 ngày và nó phân rã cho bức xạ hạt beta. Nếu cần 10,0µg
Ca cho một thí nghiệm, khối lượng của 47CaSO4 tối thiểu cần mua là bao nhiêu nếu nó mất 48 giờ để chuyển
47

đến phòng thí nghiệm từ người bán?


3. Một mẫu natri photphat Na3PO4 nặng 54,5 mg chứa đồng vị phóng xạ P-32 (có khối lượng 32,0 u).
Nếu 15,6% số nguyên tử photpho trong hợp chất là P-32 (còn lại là photpho có trong tự nhiên), có bao nhiêu
hạt nhân P-32 phân rã trong một giây đối với mẫu này? P-32 có thời gian bán phản ứng là 14,3 ngày. Cho biết
P tự nhiên có khối lượng nguyên tử trung bình là 30,97 u;

Câu 4 (2,0 điểm) Nhiệt hóa học

Trộn cẩn thận 1 mol hiđro và 0,5 mol oxi ở áp suất 101,3 kPa và nhiệt độ 291K trong một xi-lanh làm
bằng thép dày, có piston.

19
1. Xác định biến thiên entanpi, entropy, năng lượng tự do Gibbs của quá trình trộn khí trên.
2. Giá trị nào (ΔH, ΔS, ΔG) sẽ thay đổi nếu quá trình được tiến hành ở nhiệt độ 273K?
3. Hỗn hợp tạo thành trong xi-lanh được nén nhanh (nhiệt trao đổi với môi trường không đáng kể) tới thể
tích 3,0 lít, sau đó được kích nổ.
a) Tính nhiệt độ của hỗn hợp ở thời điểm bắt đầu nổ (coi quá trình nén thuận nghịch).
b) Tại sao phản ứng tạo thành nước lại đi kèm với một vụ nổ?
c) Tính nhiệt độ khí đạt tới sau thời điểm xảy ra nổ và đốt cháy hoàn toàn hiđro, biết thể tích khí trong
xi-lanh tăng lên ngay lập tức do sự tăng nhiệt độ của hỗn hợp đã đẩy piston lên, tạo ra áp suất không đổi 3,2
MPa.
d) Có thể tính nhiệt độ ở ý c)với giả sử rằng Cp hầu như không phụ thuộc vào nhiệt độ và sử dụng giá
trị Cp ở 298K được không?
Cho biết: Cp,H2O(h) = 30,13 + 11,3.10-3T; ∆fHH2O(h) = 241,8 kJ.mol-1.
Câu 5 (2,0 điểm) Cân bằng hóa học trong pha khí

1. Khí NO kết hợp với hơi Br2 tạo ra một khí duy nhất trong phân tử có 3 nguyên tử.
a) Viết phương trình phản ứng xảy ra.
b) Xét tại 25oC, CBHH đã được thiết lập.
Cân bằng đó sẽ chuyển dịch như thế nào? Nếu:
- Tăng lượng khí NO.
- Giảm lượng hơi Br2.
- Thêm khí N2 vào hệ mà áp suất chung của hệ không đổi (P = const).
2. Nitơ đioxit là một trong số các oxit của nitơ được tìm thấy ở trong khí quyển. Nó có thể đime hóa
cho N2O4 (k) : 2 NO2 (k ) N2O4 (k )
a) Tại 298K, ∆G° tạo thành của N2O4(k) là 98,28 kJ, còn của NO2(k) là 51,84 kJ. Bắt đầu với 1,0 mol
N2O4 (k) tại 1,0 atm và 298K, tính % N2O4 bị phân hủy nếu áp suất tổng không đổi tại 1,0 atm và nhiệt độ
được giữ nguyên 298K.
b) Nếu ∆H° của phản ứng N2O4 (k ) 2 NO2 (k ) là 58,03 kJ, tại nhiệt độ nào % N2O4 phân hủy sẽ gấp
đôi ở phần 1.

Câu 6 (2,0 điểm) Động học hình thức

1. Ở 4530C sự đồng phân hoá cis - trans của đimetyl xiclopropan là phản ứng thuận nghịch bậc Thành phần
phần trăm của hỗn hợp phản ứng theo thời gian thu được như sau:
t (s) 0 45 90 225 270 360 495 675 
trans (%) 0 10,8 18,9 37,7 41,8 49,3 56,5 62,7 70
Tính hằng số cân bằng và hằng số tốc độ của phản ứng thuận nghịch.

20
2. Sự phân hủy NH3 thành N2 và H2 được tiến hành trên bề mặt volfram (W) có năng lượng hoạt hóa Ea =
163kJ.mol-1. Khi không có mặt xúc tác, Ea = 335 kJ.mol-1.
a) Phản ứng trên bề mặt W ở 298K nhanh hơn khoảng bao nhiêu lần so với phản ứng không có xúc tác?
[NH3 ]
b) Tốc độ phản ứng phân hủy NH3 trên bề mặt W tuân theo qui luật động học có dạng: v = k ; trong
[H2 ]
đó, k là hằng số tốc độ phản ứng; [NH3], [H2] là nồng độ của NH3 và H2. Vì sao tốc độ phản ứng lại tỉ lệ nghịch
với nồng độ của H2?
Câu 7 (2,0 điểm) Dung dịch và phản ứng trong dung dịch
A là dung dịch gồm H3PO4 0,02 M và C6H5COOH 0,06 M. B là dung dịch NH3 0,12 M.
a) Tính pH của dung dịch A và dung dịch B.
b) Trộn 10,0 mL dung dịch A với 10,0 mL dung dịch B, thu được 20,0 mL dung dịch C.
Xác định thành phần giới hạn của dung dịch C và tính pH của dung dịch C.
c) Trộn 20,0 mL dung dịch C với 10,0 mL dung dịch HCl 0,020 M và 10,0 mL dung dịch MgCl2 0,02 M thu
được hỗn hợp D. Cho biết có kết tủa NH4MgPO4 và Mg(OH)2 hình thành hay không?
Cho biết: pKa1(H3PO4) = 2,15; pKa2(H3PO4) = 7,21; pKa3(H3PO4) = 12,32; pKa(C6H5COOH) = 4,2; pKa(NH4+)
= 9,24; pKs(NH4MgPO4) =12,8; pKs(Mg(OH)2) = 10,9;
Bỏ qua sự tạo phức hydroxo của Mg2+.

Câu 8 (2,0 điểm) Phản ứng oxi hóa khử. Pin điện và điện phân

Các phản ứng oxihoá - khử cho phép đo được các số liệu nhiệt động quan trọng.
Cho sẵn các thông tin sau:
Ag+(dd) + e– → Ag(r) E° = 0,7996 V
AgBr(r) + e– → Ag(r) + Br –(dd) E° = 0,0713 V
ΔGf°(NH3(dd)) = – 26.50 kJ.mol-1
ΔGf°(Ag(NH3)2+(dd)) = – 17.12 kJ.mol-1
+1.441
V
+1.491 V +1.584 V ?
BrO3–(dd) ⎯⎯⎯⎯⎯ → HOBr ⎯⎯⎯⎯⎯ → Br2(dd) ⎯ ⎯→ Br –(dd)
1. Tính ΔGf°(Ag+(dd)).
2. Tính trị số Ksp của AgBr (r) tại 25oC.
3. Một nguyên tố ganvani dùng điện cực hidro chuẩn làm anot được xây dựng sao cho trong pin
xảy ra phản ứng sau:
Br2(l) + H2(k) + 2 H2O(l) → 2 Br –(dd) + 2 H3O+(dd).
Ion bạc được thêm cho đến khi AgBr kết tủa tại catot và [Ag+] đạt tới 0,060 M. Điện áp đo được
là 1,721 V. Tính ΔE° cho nguyên tố ganvani.

21
4. Tính độ tan của brom trong nước để tạo thành nước brom tại 25oC.
Câu 9 (2,0 điểm) Halogen, oxi, lưu huỳnh

1. Cho sơ đồ chuyển hóa các hợp chất của nguyên tố A như sau:

Xác định các chất A,B,… (không cần giải thích) và viết các PTHH.

C phản ứng với G tạo ra H và B. H bị thủy phân tạo ra D và axit mạnh I. Xác định H và I, viết các PTHH.

2. Xét sự phân hủy của đơn chất A, người ta làm thí nghiệm như sau:

Cho 3,2g đơn chất A vào một bình không có không khí, dung tích 1 lít. Đun nóng để A bay hơi
hoàn toàn. Kết quả theo dỗi nhiệt độ và áp suất được ghi lại trong bảng sau:

Nhiệt độ (oC) Áp suất (atm)

444,6 0,73554

450 0,88929

500 1,26772

900 4,80930

1500 14,53860

Xác định thành phần định tinh hơi A tại các nhiệt độ và giải thích?

3. Nguyên tố A tạo hợp chất nguyên tố với Flo A1, A2, A3 với tỉ lệ khối lượng lưu huỳnh:

Hợp chất A1 A2 A3

Phần trăm A 45,71 29,63 21,92

Xác định A1, A2, A3 (không cần giải thích), vẽ cấu trúc 3 chiều của A1 và giải thích cấu trúc này không
tương đương với tất cả nguyên tử flo

So sánh nhiệt độ sôi A2 và A3, giải thích.

Câu 10 (2,0 điểm) Đại cương hữu cơ (quan hệ giữa cấu trúc và tính chất)

22
1. Trong phản ứng Diels-Alder của dẫn xuất cyclopentadiene có thể thu được 2 sản phẩm như sau:

a) Giải thích tại sao dạng syn lại thường chiếm ưu thế, đặc biệt trong trường hợp R là những nhóm thế
lớn?

b) Tuy nhiên trong 1 số trường hợp, dạng anti lại chiếm 100%, ví dụ trong trường hợp phản ứng của
acetoxycyclopentadiene với ethylene. Giải thích trường hợp này?

2) Giải thích sự khác nhau về nhiệt độ sôi (Ts) của 2 chất sau:

CH3COOH CF3COOH

A, Ts = 118,1oC B, Ts = 72,4oC

----------------------
KỲ THI HỌC SINH GIỎI CÁC TRƯỜNG THPT CHUYÊN
KHU VỰC DUYÊN HẢI VÀ ĐỒNG BẰNG BẮC BỘ
LẦN THỨ XIII, NĂM 2022
ĐỀ THI MÔN: HÓA HỌC - LỚP 10
Thời gian làm bài: 180 phút
(Đề thi gồm 10 câu, 6 trang)
(ĐỀ GIỚI THIỆU)

Câu 1. (2,0 điểm) Cấu tạo nguyên tử, phân tử, định luật tuần hoàn
1. Sử dụng mô hình VSEPR dự đoán hình học của các chất sau:
ClO2+, ClO2-, SOCl2, XeF2, ICl2+, SbF52-, IF4-, PF4-, ICl4+, SnF62-.
2. Electron π của liên kết đôi trong anken được xem như electron chuyển động tự do trong giếng thế
hai chiều. Biểu thức tính năng lượng của electron có dạng:

h2  n 2x n 2y 
E=  2 + 2 
8m  Lx Ly 
Biết: Lx, Ly là chiều dài mỗi cạnh của giếng thế; nx, ny là số lượng tử chính của electron, là các số
nguyên dương, không phụ thuộc vào nhau; m là khối lượng electron; h là hằng số Planck.
Xét một electron chuyển động trong một giếng thế hai chiều có Lx = 8,00 nm, Ly = 5,00 nm.
a) Cho biết giá trị các số lượng tử chính của electron này ứng với ba mức năng lượng thấp nhất đầu tiên.
23
b) Tính bước sóng λ của bức xạ cần thiết để kích thích electron từ trạng thái kích thích đầu tiên lên trạng
thái kích thích thứ 2.
Câu 2. (2,0 điểm) Tinh thể
Kim loại M trong tự nhiên chủ yếu chỉ tồn tại dưới dạng hợp chất, chủ yếu là khoáng vật ortho silicate
với công thức chung là Mx(SiO4)y, và còn ở dưới dạng oxit. Oxit của nó có nhiều dạng thù hình và thường có
kiểu mạng tinh thể đơn tà biến dạng với số phối trí CNmetal = 7. Ở nhiệt độ cao hơn 1100°C thì mạng tinh thể
của nó chuyển sang cấu trúc tứ phương. Ở nhiệt độ trên 2000°C thì nó lại mang cấu trúc lập phương biến
dạng. Cấu trúc mạng lưới của kiểu tinh thể sau cùng tương tự như kiểu florit, trong đó ion kim loại tạo ô mạng
lập phương tâm diện với hằng số mạng a0 = 507 pm. Anion oxit chiếm các hốc tứ diện. Cấu trúc này có thể
được bền hóa ở nhiệt độ phòng bằng cách sử dụng CaO. Khối lượng riêng của oxit kim loại tinh khiết (cấu
trúc lập phương) trong câu hỏi này là 6.27 g·cm-3.
1. Vẽ cấu trúc ô mạng cơ sở của oxit này. Cho biết công thức thực nghiệm của oxit.
2. Cho biết số oxy hóa của kim loại trong oxit.
3. Số oxy hóa của kim loại trong oxit thì đồng nhất với số oxy hóa của kim loại trong khoáng silicat.
Vậy công thức thực nghiệm của silicat là?
4. Ở trên đã đề cập đến kim loại nào? Tính toán chứng minh.
5. Viết cấu hình electron của kim loại. Cho biết số phối trí của cation và anion trong oxit.
Câu 3. (2,0 điểm) Phản ứng hạt nhân
210
Poloni ( 84 Po ) thuộc họ phóng xạ urani - radi có chu kỳ bán rã 138,38 ngày.
210 238
1. Tính khối lượng 84 Po có trong 1kg urani tự nhiên. Cho chu kỳ bán rã của 92 U bằng 4,47.109 năm
238
và 92 U chiếm 99,28% khối lượng của urani tự nhiên.

2. Hạt nhân
210
84 Po phân rã , tạo thành đồng vị bền 206
82 Pb . Cho rằng hạt nhân
210
84 Po đứng yên, năng lượng

phân rã chuyển hóa hoàn toàn thành động năng của hạt nhân chì và hạt , làm cho hạt nhân
206
82 Pb chuyển động

giật lùi với vận tốc vL, còn hạt  chuyển động về phía trước với vận tốc v . Biết khối lượng mol của
210
84 Po
206
bằng 209,982864 g.mol-1; của 82 Pb bằng 205,974455 g.mol-1, của 24 He bằng 4,00260325 g.mol-1. Tính tốc độ
đầu của hạt  với độ chính xác đến hai chữ số có nghĩa.

3. Là nguồn phát  mạnh,


210
84 Po đã được đặt trong các tàu tự hành đổ bộ lên Mặt Trăng để tạo ra nguồn
cung cấp năng lượng sưởi ấm các thiết bị trong những đêm Mặt Trăng lạnh giá. Tính công suất phát nhiệt ban

đầu (ra Watt) của một nguồn chứa 1 g


210
84 Po . Cho rằng 100% động năng của các hạt  được hấp thụ để chuyển
thành nhiệt.
4. Tính công suất phát nhiệt trung bình (J/s) trong thời gian 138,38 ngày của nguồn ban đầu chứa 1g
210
84 Po .

24
Câu 4. (2,0 điểm) Nhiệt hóa học
Một mẫu 3,75 mol khí lý tưởng đơn nguyên tử ở 250C và 4,5 bar được chuyển theo các quá trình sau:
1
1. Giãn đoạn nhiệt thuận nghịch tới áp suất cuối cùng bằng áp suất ban đầu.
3
1
2. Giãn đoạn nhiệt chống lại áp suất ngoài 1,5 bar tới áp suất cuối cùng bằng áp suất ban đầu.
3
1
3. Giãn với áp suất ngoài bằng 0 tới áp suất cuối cùng bằng áp suất ban đầu.
3
Hãy tính Q, W, ∆U, ∆H và ∆S của mỗi quá trình trên.
Câu 5. (2,0 điểm) Cân bằng hoá học trong pha khí
Khi đun nóng, COCl2 bị phân hủy theo phản ứng:
COCl2(k) CO(k) + Cl2(k).
Ở trạng thái cân bằng, độ phân li của COCl2 là , áp suất của hệ là P.
1. Thiết lập biểu thức tính KP theo độ phân li α và áp suất P.
2. Ở 600oC và 1,38 bar, độ phân li bằng 0,9. Tính KP, KC và Kx của phản ứng ở điều kiện này.
3. Cho biết chiều hướng diễn biến của phản ứng ở 600oC trong mỗi trường hợp sau:
PCOCl2 (bar) PCO (bar) PCl2 (bar)
Trường hợp 1 1,013 1,013 1,013
Trường hợp 2 1,046 2,027 3,036
Trường hợp 3 1,048 3,039 3,039
4. Biết nhiệt hình thành của các chất: f Ho298,COCl2 = -242,61kJ / mol; f Ho298,CO = -110,53kJ / mol và
giả sử nhiệt của phản ứng không phụ thuộc vào nhiệt độ. Cho biết chiều hướng diễn biến của phản ứng ở
650oC trong các trường hợp ở ý c).
5. Cho biết chiều hướng chuyển dịch cân bằng khi:
- Thay đổi áp suất của hệ phản ứng.
- Thay đổi nhiệt độ của hệ phản ứng.
- Giữ áp suất và nhiệt độ của hệ không đổi, thêm vào hệ một lượng khí Ne.
- Giữ thể tích và nhiệt độ của hệ không đổi, thêm vào hệ một lượng khí Ne.
Câu 6. (2,0 điểm) Động hóa học hình thức
Bằng cách oxy hóa glucozơ trong thực phẩm thì oxy sẽ bị khử thành nước nhưng một lượng nhỏ lại bị
khử tạo thành gốc tự do O2-. Để huỷ diệt gốc tự do nguy hiểm này thì enzym superoxiddismutaza SOD đóng
vai trò quan trọng. Enzym này được ký hiệu là E và xúc tác cho phản ứng sau:

Người ta khảo sát phản ứng này trong một dung dịch đệm có pH = 9.1. Nồng độ đầu của SOD có giá trị [E]0
= 0,400·10-6 M. Tốc độ đầu v0 của phản ứng trên được đo ở nhiệt độ phòng bằng cách sử dụng những nồng
độ đầu khác nhau của anion gốc tự do O2- .

25
C0(O2-) mol/L 7.69·10-6 3.33·10-5 2.00·10-4
v0 mol/L·s 3.85·10-3 1.67·10-2 0.100
1. Xác định bậc phản ứng ứng với biểu thức tốc độ v = k·[O2-]n.
2. Xác định hằng số tốc độ k.
Phản ứng trên có cơ chế được đề nghị như sau:

3. Xây dựng biểu thức tốc độ cho cơ chế này, cho rằng k2>k1. Xác định xem biểu thức tốc độ mới có
khớp với biểu thức tốc độ ở câu 4.1 hay không.
Biết E- rất không bền cho nên [E-] là hằng định trong một khoảng thời gian ngắn.
4. Sử dụng nguyên lý nồng độ dừng cho E -, hãy tính giá trị hai hằng số tốc độ k1 và k2 nếu biết rằng
k2 lớn gấp đôi k1.
Câu 7. (2,0 điểm) Dung dịch và phản ứng trong dung dịch
Cho dung dịch A là dung dịch H3PO4 0,020M; dung dịch B là dung dịch Na3PO4 0,010M.
1. Tính nồng độ cân bằng của các cấu tử trong dung dịch A.
2. Cho từ từ 10,00 ml dung dịch A vào 10,00 ml dung dịch B thu được 20,00 ml dung dịch C. Tính pH
của dung dịch C.
3. Cho từ từ Na3PO4 vào dung dịch chứa CdCl2 0,010M và ZnCl2 0,010M. Coi thể tích dung dịch thay
đổi không đáng kể trong quá trình thí nghiệm.
a) Kết tủa nào xuất hiện trước?
b) Khi kết tủa thứ hai xuất hiện thì nồng độ của ion thứ nhất còn lại bao nhiêu? Từ đó đánh giá khả
năng dùng dung dịch Na3PO4 để tách riêng 2 ion Cd2+ và Zn2+ ra khỏi nhau từ dung dịch gồm CdCl2 0,010M
và ZnCl2 0,010M. Biết rằng 2 ion được coi là tách ra khỏi nhau nếu ion thứ nhất kết tủa hết (tổng nồng độ các
dạng tồn tại trong dung dịch  10-6M) thì ion thứ 2 chưa kết tủa. Cho biết:

Câu 8. (2,0 điểm) Phản ứng oxi hoá khử. Pin điện và điện phân
1. Tính thế khử chuẩn của cặp Fe3+/Fe2+ trong môi trường axit và thế khử chuẩn của cặp Fe(OH)3/Fe(OH)2
trong môi trường kiềm. Khả năng khử của Fe(II) trong môi trường nào mạnh hơn?
Cho biết: EFe
o
2+
/Fe
= −0, 440 V; EFe
o
3+
/Fe
= −0,036 V ; pKs(Fe(OH)2) = 14,78; pKs(Fe(OH)3) = 37,42.

2. Thêm V (mL) dung dịch K2Cr2O7 0,02 M vào 100 mL dung dịch FeSO4 0,12 M (tại pH = 0 và không
đổi trong suốt quá trình phản ứng), thu được dung dịch A. Tính thế khử của cặp Fe3+/Fe2+ trong dung dịch A ở
mỗi trường hợp sau đây: i) V = 50 mL; ii) V = 100 mL; iii) V = 101 mL.
Cho biết: EFe
o
3+
/Fe2+
= 0,771 V; ECro O2− ,H+ /2Cr3+ = 1,330 V.
2 7

Câu 9. (2,0 điểm) Halogen, Oxi – lưu huỳnh

26
Đun nóng hỗn hợp lưu huỳnh và AgF ở 125 °C thu được khí A1 có tỉ khối hơp so với heli bằng 25,5.
Khi đun nóng A1 phân hủy tạo thành lưu huỳnh và khí A2. Đun nóng A2 với ClF ở 380 °C được khí A3 chứa
ba loại nguyên tố. A3 bị khử quang hóa với H2 tạo ra chất lỏng A4 không phân cực, không chứa clo. Phần
trăm khối lượng lưu huỳnh trong A2, A3 và A4 lần lượt là 29,630 %; 19,692 % và 25,197 %. Mỗi phân tử A2
và A3 chỉ chứa một nguyên tử lưu huỳnh.
a) Xác định và vẽ cấu tạo của A1, A2, A3 và A4.
b) Viết phương trình hóa học khi đun nóng từng chất A1, A2, A3 và A4 trong dung dịch NaOH đặc.
Câu 10. (2,0 điểm) Đại cương hữu cơ (quan hệ giữa cấu trúc và tính chất)
1. So sánh tính bazơ của các hợp chất sau và giải thích:
CH3-CH(NH2)-COOH (I) ; CHC-CH2-NH2 (II) ; CH2=CH-CH2-NH2 (III) ; CH3-CH2-CH2-NH2 (IV).
2. So sánh (có giải thích) tính bazơ của các hợp chất A và B dưới đây:

N
C6H5-CHOH-CH2NH- C6H5-CHOH-CH2NH-
A N
B
3. So sánh (có giải thích) tính bazơ của hai hợp chất X, Y dưới đây:

4. Viết công thức cấu tạo của các chất sau đây và sắp xếp theo thứ tự tăng dần tính bazơ: pyridin, các
aminopyridin, 3-clopyridin, 3-nitropyridin và giải thích ngắn gọn.
-----------------Hết----------------
SỞ GIÁO DỤC VÀ ĐÀO TẠO KỲ THI HỌC SINH GIỎI CÁC TRƯỜNG THPT CHUYÊN
TỈNH BÌNH DƯƠNG KHU VỰC DUYÊN HẢI VÀ ĐỒNG BẰNG BẮC BỘ
TRƯỜNG THPT CHUYÊN LẦN THỨ IX, NĂM 2022
HÙNG VƯƠNG
ĐỀ THI MÔN: HÓA HỌC - KHỐI 10
Thời gian: 180 phút.
ĐỀ THI ĐỀ XUẤT Đề thi gồm có: 10 câu, 06 trang.

27
Học sinh không được sử dụng bảng tuần hoàn các nguyên tố hoá học.
Các kí hiệu viết tắt: r: rắn, l: lỏng, k: khí, aq: dung dịch, oxh: oxy hoá, f: tạo thành, đktc: điều kiện tiêu chuẩn,
t-Bu: tert-butyl.
Các hằng số, phương trình và đơn vị quy đổi:
Hằng số Avogadro: NA = 6,022.1023
Hằng số khí lí tưởng: R = 0,082 atm.L.K-1mol-1 = 8,314 J.K-1.mol-1
Hằng số Faraday: F = 96485 C.mol-1
Áp suất chuẩn: 𝑝𝑜 = 1 bar =105 Pa
Tốc độ ánh sáng: c = 2,998.108 m.s-1
Điện tích cơ bản: е = 1,602.10-19 C
Hằng số điện: 𝜀𝑜 = 8,854.10-12 F/м
Số Pi: 𝜋 = 3,14
Định luật Bragg: 𝑛. 𝜆 = 2𝑑. 𝑠𝑖𝑛 𝜃
Nhiệt độ tuyệt đối: T (K) = t (oC) + 273
Biến thiên nội năng: 𝑑𝑈 = 𝛿𝑄 + 𝛿𝑊
Công thể tích: 𝛿𝑊 = −𝑝𝑑𝑉
𝑅𝑇 [𝑜𝑥ℎ] 𝑅𝑇 0,0592
Phương trình Nernst: 𝐸 = 𝐸 𝑜 + 𝑛𝐹 𝑙𝑛 [𝑘ℎử] với 𝑛𝐹 𝑙𝑛 = lg
𝑛

Electron-volt: 1 eV = 1,602.10-19 J
Watts: 1 W = 1 J/s
Khối lượng nguyên tử: 1 amu = 1,6605.10-27 kg = 931,5 MeV/c2
Thể tích mol khí lý tưởng ở đktc: 22,4 dm3/mol (tại 0 oC và 1 atm).
Cho khối lượng nguyên tử của các nguyên tố (g/mol):
Cs = 132,91; Cl = 35,45; Cr = 52; K = 39; O = 16; Mn = 55; Br = 80; C = 12,01; N = 14,01; H = 1,008; I =
127,1; S = 32; Fe = 56; P = 31.
Các khí và hơi được xem như khí lý tưởng.
Chấp nhận hoạt độ của các các tiểu phân trong dung dịch nước xấp xỉ bằng nồng độ (M).
Câu 1: (2.0 điểm) Cấu tạo nguyên tử, phân tử, định luật tuần hoàn.
1.1. Năm 1913, Bohr đã phát triển mô hình của ông về nguyên tử hydrogen. Mô hình này dựa trên giả thiết
rằng nguyên tử có các quỹ đạo tròn ổn định, trong đó các electron có vị trí xác định mà không bức xạ năng
lượng. Electron chuyển từ quỹ đạo n1 đến n2 sẽ kéo theo việc hấp thụ hoặc bức xạ ánh sáng (photon) có bước
sóng xác định. Thế năng của electron trong trường tĩnh điện của hạt nhân là:
𝑒2
𝐸𝑛 = −
4𝜋𝜀𝑜 𝑟𝑛

28
Trong đó: 𝑒 – điện tích cơ bản, 𝜋 – số Pi, 𝜀𝑜 – hằng số điện, 𝑟𝑛 – bán kính của orbital thứ n và 𝑟𝑛 = 𝑎𝑜 𝑛2 với
𝑎𝑜 – bán kính của quỹ đạo Bohr thứ nhất (bán kính Bohr).
Cho biết giá trị động năng của nguyên tử hydrogen chỉ nhỏ bằng ½ và ngược dấu với thế năng của nó. Năng
lượng của nguyên tử hydrogen là −2,189.10–18 J.
a. Tính bán kính Bohr 𝑎𝑜 và khoảng cách cực tiểu giữa các quỹ đạo thứ 4 và thứ 3 (theo pm).
Cho biết năng lượng tổng của phân tử H2 là −3070 kJ/mol (năng lượng được đo từ năng lượng của hạt nhân
và các electron ở trạng thái nghỉ, nằm ở những vị trí cách xa nhau vô tận).
b. Tính năng lượng liên kết giữa các nguyên tử hydrogen trong phân tử này.
1.2. Giả thiết ở một vũ trụ khác, các định luật vật lý được đặt lại khác với ở vũ trụ của chúng ta. Bảng hệ thống
tuần hoàn vì vậy được sắp xếp theo một trật tự khác. Các số lượng tử n, l, ml, ms giờ đây được gọi là m, n, p, q
tương ứng. Quy luật của chúng như sau:
- m nhận các giá trị số nguyên dương (m > 0): 1, 2, 3, 4, …
- n nhận các giá trị số nguyên dương chẵn nằm trong đoạn [0, m] (số không được coi là số chẵn).
- p nhận các giá trị số nguyên lẻ. Với p dương thì n  p  2n, với p âm thì -2n  p  -n.
1
- q nhận hai giá trị là ± 2.

Bằng lập luận, hãy cho biết có bao nhiêu nguyên tố có m = 6 ?


Câu 2: (2.0 điểm) Tinh thể.
Mạng lưới Bravais xuất hiện do sự tịnh tiến một điểm theo ba phương khác nhau nên tất cả các điểm
mạng đều đồng nhất. Nếu tinh thể chỉ chứa một loại nguyên tử A thì mỗi nguyên tử này có thể được coi là một
điểm mạng của một mạng lưới Bravais tương ứng. Nếu tinh thể chứa nhiều loại nguyên tử A, B, C… khác
nhau thì ứng với mỗi loại nguyên tử có một mạng lưới Bravais và những mạng lưới này lồng vào nhau tạo
thành mạng lưới tinh thể chung (mạng lưới Bravais kép).
Cl-

Cs+

Như trong tinh thể trên, các ion Cs+ và Cl- tạo thành một mạng lưới Bravais lập phương đơn giản và hai mạng
lưới này lồng vào nhau sao cho đỉnh của các tế bào do cùng một loại ion tạo nên nằm ở trọng tâm các tế bào
bạo bởi loại ion khác dấu.
2.1. Xác định công thức thực nghiệm của tinh thể trên và cho biết số phối trí của ion Cs+.
Người ta tiến hành nhiễu xạ tia X đơn tinh thể trên, sự phản xạ bậc nhất của mặt phẳng (100) được quan sát tại
góc tới là 10,78°.
29
2.2. Hãy tính khối lượng riêng của tinh thể này, biết rằng chùm tia X này có bước sóng là 1,542 Å.
2.3. Tính bán kính của ion Cs+, biết rằng các ion trái dấu tiếp xúc với nhau tại đường chéo của ô mạng cơ sở
và bán kính ion Cl- là 1,81 Å.
Câu 3: (2.0 điểm) Phản ứng hạt nhân.
Nguyên tố Thorium được phát hiện lần đầu tiên vào năm 1828 và sau đó nó được xác định và đặt tên
theo Thor, thần sấm trong thần thoại Bắc Âu. Trong tự nhiên, Thorium là kim loại phóng xạ, và cũng được
xem là một nguyên liệu hạt nhân thay thế cho Uranium. Một chuỗi phóng xạ tự nhiên bắt đầu từ 232
90Th (𝑡1/2 =

1,40. 1010 năm) và kết thúc với đồng vị bền là 208


82Pb.

3.1. Bằng tính toán, hãy cho biết có bao nhiêu phân rã beta (𝛽 − ) có trong chuỗi phóng xạ trên.
3.2. Tính năng lượng theo MeV được giải phóng ra từ một chuỗi phóng xạ trên.
3.3. Tính công suất theo Watts khi 1,00 kg 232Th phân rã trong chuỗi phóng xạ trên.
Một sản phẩm trong chuỗi phóng xạ trên là 228Th (𝑡1/2 = 1,91 năm). Cho rằng chu kì bán huỷ của tất cả các
hạt nhân nguyên tử trung gian đều nhỏ hơn rất nhiều so với chu kì bán huỷ của 228Th.
3.4. Tính thể tích khí helium theo cm3 ở 20 °C và 1 atm thu được khi lưu trữ 1,00 g 228Th trong một bình chứa
trong suốt 20,0 năm.
3.5. Sau khi phân lập một sản phẩm khác trong chuỗi Thorium, thu được 1,50.1010 nguyên tử. Tính chu kì
bán huỷ theo năm của sản phẩm này, biết rằng sản phẩm này phân rã với tốc độ 3440 phân rã mỗi phút.
Cho biết khối lượng nguyên tử của:
4
2𝐻𝑒 = 4,00260 𝑎𝑚𝑢, 208 232
82𝑃𝑏 = 207,97664 𝑎𝑚𝑢, 90𝑇ℎ = 232,03805 𝑎𝑚𝑢.

Câu 4: (2.0 điểm) Nhiệt hoá học.


Một mol khí chlorine ban đầu ở 300K và 100 atm được giãn nở chống lại áp suất bên ngoài là png = 1
atm đến áp suất cuối bằng với áp suất bên ngoài. Kết quả của sự giãn nở là hệ được làm lạnh đến nhiệt độ 239K
(cũng chính là điểm sôi thông thường của Cl2), quan sát thấy có 0,100 mol khí Cl2 ngưng tụ.
4.1. Tính biến thiên nội năng của hệ.
4.2. Tính biến thiên entropy cho cả quá trình trên.
Cho biết:
+ Ở 239K, Cl2(l) có nhiệt hoá hơi là 20,42 kJ/mol và có khối lượng riêng bằng 1,56 g/cm3.
+ 𝐶𝑣,𝐶𝑙2(𝑘) = 28,66 J.K-1.mol-1 và Cp = Cv + R.

Câu 5: (2.0 điểm) Cân bằng hoá học trong pha khí.
Ở 560 K, áp suất của bình phản ứng có thể tích không đổi chỉ chứa hỗn hợp CuBr2(r), CuBr(r) và Br2(k) là
1
1,00 bar. Chấp nhận rằng ∆𝐻 𝑜 và ∆𝑆 𝑜 của phản ứng: CuBr2(r) ⟶ CuBr(r) + 2Br2(k) không phụ thuộc vào nhiệt

độ và bỏ qua thể tích của các chất rắn, hãy tính:


5.1. Áp suất của bình phản ứng trên ở 570 K.
30
𝑜
5.2. Entropy chuẩn 𝑆298𝐾 của CuBr2(r).
Cho 1,000 mol CuBr2 rắn vào một bình kín, ban đầu không chứa chất nào khác. Nhiệt độ của bình được duy
trì cố định ở 570 K.
5.3. Xác định thành phần của hệ (theo số mol) và áp suất (theo bar) của bình phản ứng tại 𝑡 = +∞ khi thể tích
của bình phản ứng là: a. 5,00 L và b. 20,00 L.
𝑜
Cho biết: Chất ∆𝑓 𝐻298𝐾 ( kJ/mol) 𝑜
𝑆298𝐾 (J.K-1.mol-1)
CuBr2(r) −138,9 ?
CuBr(r) −105,0 91,60
Br2(k) 30,7 245,35
Câu 6: (2.0 điểm) Động hoá học hình thức.
k1
Cho phản ứng cộng hợp: A + B AB
Động học của phản ứng đã được nghiên cứu bằng sắc kí với nồng độ đầu của các chất phản ứng bằng nhau.
Bảng 6.1 cho biết sự phụ thuộc của nồng độ sản phẩm AB theo thời gian.
Bảng t, phút 0 20 40 60 80 100 +∞ 6.1:

CAB, M 0 0,111 0,171 0,208 0,234 0,252 0,370


6.1. Xác định bậc của phản ứng tổng và hằng số tốc độ k1. Biết rằng bậc của phản ứng có thể là số nguyên
hoặc bán nguyên.
Ở nhiệt độ cao, ngoài sản phẩm cộng, còn diễn ra phản ứng song song là sự dimer hoá A với hằng số tốc độ k2:
k2
Phản2A
ứng dimer hoáAcó
2 bậc giống với phản ứng cộng. Bảng 6.2 cung cấp các giá trị tốc độ tiêu thụ tổng của A
trong 2 phản ứng ở hai thời điểm khác nhau.
Bảng 6.2: Tốc độ tiêu thụ tổng của A,
t, phút CA , M CB , M CAB , M CA2 , M
vA, M/phút
0 0,2630 0,3700 0,3700 0 0
3 0,0296 0,1020 0,2670 ? ?

6.2. Tìm CAB , CA2 tại thời điểm t = 3 phút và tính các giá trị k1, k2.
Tốc độ tiêu thụ A có thể biểu diễn theo định luật tác dụng khối lượng kinh nghiệm: vA = kexp.(𝐶𝐴 )𝑥
6.3. Hãy dự đoán giới hạn khả thi của x. Sau đó sử dụng dữ kiện trong Bảng 6.2, hãy xác định bậc kinh
nghiệm x của A trong 3 phút đầu tiên của phản ứng.
Câu 7: (2.0 điểm) Dung dịch và phản ứng trong dung dịch.
7.1. Tính pH của dung dịch A chứa H2SO4 0,080 M và Fe2(SO4)3 0,100 M.
7.2. Chứng minh rằng không thể hoà tan 0,1 mol CuS bằng dung dịch HCl 1,000 M nhưng có thể hoà tan nó
bằng dung dịch HNO3 1,000 M. Cho rằng ion Cu2+ không tạo phức bền với ion Cl- trong điều kiện trên, quá
trình hoà tan này diễn ra ở 25 °C và không làm thay đổi thể tích dung dịch.

31
Cho biết: pKa (HSO4-) = 1,99; *𝛽𝐹𝑒(𝑂𝐻)2+ = 10−2,17; pKs (Fe(OH)3) = 37,00; pKai (H2S) = 7,02; 12,9;
𝑜 𝑜
*𝛽𝐶𝑢(𝑂𝐻)+ = 10−8 ; pKs (CuS) = 35,2; 𝐸𝑆/𝑆 2− = −0,48 𝑉; 𝐸𝑁𝑂 − /𝑁𝑂
3 (𝑎𝑞)
= 0,96 𝑉.

Độ tan của NO trong nước ở 25 °C là 2,530.10-2 M.


Độ tan của H2S trong nước ở 25 °C là 0,100 M.
Câu 8: (2.0 điểm) Phản ứng oxy hoá khử, pin điện và điện phân.
8.1. Pin Galvanic đầu tiên được A. Volta chế tạo vào năm 1800, dựa vào những thí nghiệm của L. Galvani.
Sau này, các pin Galvanic đã được ứng dụng rộng rãi trong khoa học, công nghệ và cuộc sống thường ngày.
Xét một pin Galvanic có nửa bên trái chứa một điện cực sắt (dư) bị oxy hoá trong quá trình hoạt động và
dung dịch sắt (III) nitrate có nồng độ 0,010 M. Nửa bên phải của pin chứa điện cực than chì và hỗn hợp sắt
(II), sắt (III) nitrate với nồng độ lần lượt là 0,050 M và 0,300 M.
a. Điện cực nào là cathode, điện cực nào là anode? Giải thích ngắn gọn.
b. Viết sơ đồ pin, các bán phản ứng xảy ra trên mỗi điện cực và phản ứng tổng cộng.
c. Tính suất điện động ban đầu của pin và các thế ban đầu của cathode, anode ở 25oC.
Cho biết:
𝑜 𝑜
+ Ở 25oC có 𝐸𝐹𝑒 -1 -1
2+ /𝐹𝑒 = −0,441 𝑉; 𝑆𝐹𝑒(𝑟) = 27,4 J.K .mol ;

𝑜 -1 -1 𝑜 -1 -1
𝑆𝐹𝑒 2+ (𝑎𝑞) = −137,7 J.K .mol ; 𝑆𝐹𝑒 3+ (𝑎𝑞) = −316,0 J.K .mol

+ Khi tăng nhiệt độ của pin thêm 18 oC sẽ làm hằng số cân bằng K của phản ứng tổng cộng giảm 58 lần.
8.2. Việc phủ kim loại bằng chromium có thể được thực hiện bằng cách điện phân dung dịch acid chromic
(H2CrO4). Quá trình điện phân có thể được thực hiện với dòng điện 1500 A trong 7,00 giờ. Anode trơ không
thay đổi và tạo thành một khí. Ở cathode thì tạo thành sản phẩm phụ là 4,15 m3 hydrogen ở điều kiện chuẩn
(25oC; 1,00 bar), làm giảm hiệu suất dòng điện của quá trình mạ chromium.
a. Viết các bán phản ứng tương ứng với các quá trình xảy ra tại mỗi điện cực.
b. Tính hiệu suất dòng (%) của quá trình mạ chromium ở cathode.
c. Tính khối lượng chromium được mạ lên.
d. Tính thể tích khí được tạo thành tại anode ở điều kiện chuẩn.
Câu 9: (2.0 điểm) Halogen, oxygen và lưu huỳnh.
9.1. Fluorine là nguyên tố halogen hoạt động hoá học nhất, có thể phản ứng với các halogen Cl 2, Br2 và I2
trong điều kiện khống chế tỉ lệ lần lượt tạo thành hợp chất chứa bốn, sáu và tám nguyên tử.
a. Xác định công thức phân tử và vẽ cấu trúc của 3 hợp chất kể trên dựa vào cơ sở thuyết VSEPR. Biểu
diễn vị trí các cặp electron tự do (nếu có).
Phổ khối hay phổ khối lượng (Mass Spectrum) hiện nay là một phần quan trọng trong việc nghiên cứu xác
định cấu trúc phân tử. Điểm đặc trưng của phương pháp này là có thể xác định được khối lượng, chính xác hơn
là tỉ lệ khối lượng trên điện tích (m/z) của các ion tạo thành khi ion hoá phân tử và các sản phẩm phân mảnh
liên tiếp từ các sản phẩm ion hoá đó. Tiến hành đo phổ khối của một hỗn hợp gồm hơi iodine và khí chlorine,
32
thu được hai bộ giá trị (A và B) ứng với hai sản phẩm chính tạo thành từ hỗn hợp ban đầu, tại các đỉnh m/z có
cường độ cao nhất là:
A: 162, 164.
B: 464, 466, 468, 470, 472, 476.
Cho rằng các tiểu phân ứng với các giá trị m/z trên là sản phẩm ion hoá đầu tiên của mỗi phân tử, I chỉ có 1
đồng vị 127I và Cl có hai đồng vị là 35Cl và 37Cl.
b. Xác định công thức phân tử (ghi rõ số khối của nguyên tử chlorine) của các phân tử tạo thành các tiểu
phân ứng với giá trị m/z = 162, 164, 470 và 476.
c. Vẽ cấu trúc của phân tử tạo thành tiểu phân nặng nhất (có m/z = 476). Biết rằng trong cấu trúc của phân
tử có hai loại nguyên tử Cl khác nhau.
9.2. Hydrogen peroxide H2O2 phản ứng định lượng với hợp chất X (phân tử chứa 3 nguyên tố, K, O và kim
loại Y, hàm lượng K là 39,39%) trong môi trường acid thu được khí A. Cho biết cứ 2,376 g X, phản ứng với
H2O2 dư thì thu được 403,20 mL khí A (đktc). Mặt khác, khi cho H2O2 phản ứng với đơn chất Z trong huyền
phù Ca(OH)2 (vừa đủ) thì thu được 98,56 mL khí A (đktc). Cô cạn dung dịch thì thu được 1,294 g chất rắn B
duy nhất ở dạng khan. Bằng tính toán, hãy xác định các chất A, B, X, Y, Z và viết các phương trình ion của
các phản ứng xảy ra. Cho biết trong X chỉ chứa một nguyên tử kim loại Y.
Câu 10: (2.0 điểm) Đại cương hữu cơ (quan hệ giữa cấu trúc và tính chất).
10.1. So sánh sức căng vòng của các hợp chất sau. Giải thích ngắn gọn.
a. b.

10.2. Dựa vào sự hiểu biết về cấu dạng, hãy giải thích các vấn đề sau:
a. 1,2-dichloroethane có moment lưỡng cực nhỏ hơn ethylene glycol.
b. Cis-1,4-di-tert-butylcyclohexane tồn tại chủ yếu ở cấu dạng thuyền xoắn hơn là ở cấu dạng ghế.
10.3. Vẽ cấu dạng bền nhất (ở dạng ghế) của tiền chất sau để có xảy ra phản ứng:

Từ cấu dạng đó, hãy vẽ cơ chế cho phản ứng.


--------------Hết--------------

33
HỘI CÁC TRƯỜNG CHUYÊN MÔN HÓA HỌC KHỐI 10

VÙNG DUYÊN HẢI VÀ ĐỒNG BẰNG BẮC BỘ NĂM 2022

TRƯỜNG THPT CHUYÊN HÙNG VƯƠNG Thời gian làm bài 180 phút
((Đề thi có 05 trang, gồm 10 câu)
ĐỀ THI ĐỀ XUẤT

Câu 1: (2,0 điểm) Cấu tạo nguyên tử, phân tử, định luật tuần hoàn
Nguyên tử của nguyên tố X có electron cuối cùng có bộ các số lượng tử:
1
n = 2; l = 1; ml = +1 và s = + .
2

1. Viết cấu hình electron của nguyên tử nguyên tố X và xác định nguyên tố X.

2. Bằng thiết bị và ở điều kiện thích hợp, một bức xạ có độ dài sóng là 58,34 nm được chiếu vào một
dòng khí X2. Người ta xác định được tốc độ của dòng electron đầu tiên là 1,4072.106 m.s–1, tốc độ của dòng
electron tiếp theo là 1,266.106 m.s–1. Tính năng lượng ion hóa thứ nhất (I1) và năng lượng ion hóa thứ hai (I2)
của phân tử X2 theo kJ.mol–1.
3. Sử dụng thuyết obitan phân tử (thuyết MO) để giải thích tại sao năng lượng ion hóa thứ nhất của
phân tử X2 (tính được ở ý 2) lớn hơn năng lượng ion hóa thứ nhất của nguyên tử X (1420kJ/mol).
Cho: Hằng số Planck h = 6,6261.10–34 J.s; Tốc độ ánh sáng c = 2,9979.108 m.s–1; Khối lượng electron me =
9,1094.10–31 kg, số Avogađro NA = 6,0221.1023 mol–1;
4. Nguyên tố X có thể tạo được các ion mạch thẳng X5+ và mạch vòng X5-.
a. Viết công thức Lewis cho các dạng cộng hưởng của anion X5-.
b. Viết công thức Lewis cho các dạng cộng hưởng của X5+, ghi rõ trạng thái lai hóa tương ứng với mỗi
nguyên tử X, từ đó hãy cho biết dạng hình học của X5+.

Câu 2. (2,0 điểm) Tinh thể

Bạc kim loại có cấu trúc tinh thể lập phương tâm diện. Bán kính nguyên tử của Ag và Au lần lượt là:
rAg = 144 pm; rAu = 147 pm.
1. Tính số nguyên tử Ag có trong một ô mạng cơ sở.
2.Tính khối lượng riêng của bạc kim loại.
3. Một mẫu hợp kim vàng – bạc cũng có cấu trúc tinh thể lập phương tâm diện. Biết hàm lượng Au
trong mẫu hợp kim này là 10%. Tính khối lượng riêng của mẫu hợp kim.
Cho: nguyên tử khối của Ag là 108, của Au là 197.

34
Câu 3. (2,0 điểm) Phản ứng hạt nhân

1. Hãy viết phương trình biểu diễn các biến đổi hạt nhân: 98Mo tác dụng với nơtron và hạt nhân vừa
được tạo thành lại tiếp tục phân rã  tạo ra 99Tc.

2. Một mẫu 137Ce (t1/2 = 30,17 năm) có độ phóng xạ ban đầu 15,0 Ci. Hãy tính thời gian để hoạt độ
phóng xạ của mẫu này còn lại 1,50 Ci.

3. Khi bắn phá hạt nhân 235U bằng một nơtron, người ta thu được các hạt nhân

138
Ba, 86Kr và 12 hạt nơtron mới.

a. Hãy viết phương trình của các phản ứng hạt nhân đã xảy ra.

b. Tính năng lượng thu được (ra kJ), khi 2,00 gam 235U bị phân hạch hoàn toàn.

Câu 4. (2,0 điểm) Nhiệt hóa học

Ở điều kiện 250C và áp suất của hệ không đổi, tiến hành đốt cháy hoàn toàn 0,10 mol C8H18 bằng một
lượng oxi vừa đủ. Sau phản ứng thu được H2O, CO và CO2 ở 3000C và toả ra môi trường một lượng nhiệt là
90,2 kcal.

1. Tính số mol mỗi khí CO và CO2

2. Nếu 90,2 kcal nhiệt toả ra trên được tận dụng hoàn toàn vào mục đích đun sôi nước uống. Hãy tính
khối lượng nước có thể đun sôi được từ 250C đến 1000C. Cho các số liệu nhiệt động sau:

Chất C8H18 CO CO2 H2O(hơi) H2O(lỏng)

H0f (kcal/mol) -64,6 -26,41 -94,05 -57,79 …

Nhiệt dung (CP) … 8,96 8,96 5,92 18


cal/mol.K

Giả sử H0f , Cp không thay đổi theo nhiệt độ. Cho nhiệt hoá hơi của nước bằng 548 cal/gam ở 373K.

Câu 5. (2,0 điểm) Cân bằng hóa học trong pha khí
1) Hãy cho biết phản ứng 2Ni (l) + O2 (k) 2NiO (r) ở 1627 oC có thể tự diễn biến theo chiều thuận
được không nếu áp suất riêng phần của oxi nhỏ hơn 150 Pa?
Cho: G 0hình thành (NiO) ở 1627 oC là -72,1 kJ. mol–1; Áp suất chuẩn P0 = 1,000.105 Pa;
0oC trong thang Celsius là 273,15 K.
2. Người ta tiến hành tổng hợp NH3 với sự có mặt chất xúc tác Fe theo phản ứng sau:
1 3
N 2 + H 2 ⎯⎯→ NH3
2 2
35
Khi tổng hợp tỉ lệ mol N2 và H2 là 1 : 3. Trong quá trình tổng hợp chúng ta thu được các số liệu thực
nghiệm sau:

Nhiệt độ Ở Ptổng = 10 atm Ở Ptổng = 50 atm


Lượng % NH3 chiếm giữ Lượng % NH3 chiếm giữ
350oC 7,35 25,11
450oC 2,04 9,17

a. Xác định Kp theo số liệu thực nghiệm của bảng trên.


b. Tính giá trị ΔH của phản ứng ở Ptổng đã cho.

Câu 6. (2,0 điểm) Động hóa học hình thức


Hàm lượng rượu trong máu có thể được xác định bằng các định luật động học. Quá trình đào thải
rượu ra khỏi cơ thể một cách gần đúng có thể biểu diễn bằng sơ đồ sau

A ⎯⎯
k
→ B ⎯⎯
1 k
→D2

Trong đó A là rượu trong dạ dày, B là rượu trong máu, D là sản phẩm oxi hóa rượu bằng các men
→ B tuân theo quy luật phản ứng bậc một. Giai đoạn B ⎯⎯
trong gan. Giai đoạn A ⎯⎯ → D tuân theo quy
luật phản ứng bậc không.

1. Nồng độ rượu trong dạ dày giảm đi 2 lần sau 5 phút. Tính k1.

2. Chứng minh rằng biểu thức biểu diễn sự phụ thuộc nồng độ rượu trong trong máu theo thời gian có
dạng [B] = [A]o .(1 − e ) − k2t .
− k1t

3. Nếu nồng độ rượu trong dạ dày ban đầu là 3,8 g/L thì chỉ có thể nhận biết được lượng rượu trong
máu sau tối đa 20 giờ. Tính k2.

Câu 7. (2,0 điểm) Dung dịch và phản ứng trong dung dịch

Cho dung dịch X gồm H3PO4 C (mol/l) và HA 0,01 M.

1. Tính nồng độ của H3PO4 và hằng số cân bằng của axit HA, biết rằng độ điện ly của H3PO4 và HA trong
dung dịch X lần lượt là 0,443 và 1,95.10-4
2. Thêm dần dung dịch NH3 vào dung dịch X đến nồng độ 0,16 M (coi thể tích không đổi khi thêm NH3)
được dung dịch B. Tính pHB.
3. Trộn 5 ml dung dịch B với 5 ml dung dịch Mg(NO3)2 0,03 M. Bằng các phép tính cụ thể, hãy cho biết
có kết tủa tách ra không? Tính pH của hệ thu được.
Cho pKa(H3PO4)= 2,15; 7,21; 12,32; pKa(NH4+)= 9,24; pKs(MgNH4PO4) = 12,6; pKs(Mg(OH)2 = 10,9.
Câu 8. (2,0 điểm) Phản ứng oxi hóa – khử. Pin điện và điện phân

Điện cực loại II là điện cực tạo bởi kim loại được bao phủ bởi muối ít tan của kim loại đó, nhúng vào dung
dịch muối tan chứa anion của muối ít tan. Ví dụ như điện cực bạc/bạc clorua (Ag, AgCl/Cl-) và điện cực
calomen (Hg, Hg2Cl2/Cl-). Suất điện động của một tế bào điện hóa: (-) Ag,AgCl/KCl/Hg2Cl2/Hg (+) là E0 =
0,0455 V ở T = 298 K. Hệ số nhiệt độ của tế bào này là: dE0/dT = 3,38.10-4 V K-1.
36
1. Cho biết phương trình phản ứng xảy ra ở cả hai điện cực và phản ứng tổng cộng.

2 Tính năng lượng tự do Gibbs (ΔG0) cho quá trình diễn ra trong pin điện ở 298 K. Cho biết ý nghĩa dấu của
ΔGo.

3 Tính biến thiên entanpi cho quá trình ở 298 K, biết rằng ΔS = nFΔE/ΔT.

4. Biết rằng thế chuẩn của Ag/Ag+ là E0 = 0,799 V và tích số tan của AgCl là Ksp = 1,73.10-10, tính giá trị thế
điện cực chuẩn của điện cực bạc/bạc clorua. Thiết lập phương trình cho biết sự phụ thuộc giữa E0(Ag/Ag+) và
E0(Ag,AgCl/Cl-).

5. Tính tích số tan của Hg2Cl2 biết rằng, thế điện cực chuẩn của điện cực calomen là E0 = 0,798 V.

Câu 9. (2,0 điểm) Halogen, Oxi – lưu huỳnh


Cho sơ đồ chuyển hóa sau:

+ X14

Cho biết các chất từ X1 đến X14 đều là các hợp chất của lưu huỳnh (trừ X11). Biết rằng X14 là một axit
mạnh rất phổ biến trong công nghiệp, X7 và X8 có cấu trúc tương tư nhau.

Khi thủy phân hoàn toàn các chất X8 và X10, thu được các dung dịch axit và không thấy khí thoát ra. Thêm
dung dịch Ba(NO3)2 dư vào các dung dịch trên đều thu được kết tủa trắng X. Lọc kết tủa X, thêm tiếp dung
dịch AgNO3 dư vào nước lọc, lại thu được kết tủa trắng Y. Trong cả hai thí nghiệm tiến hành với hai chất X8
m 
và X10 ở trên, tỉ lệ khối lượng kết tủa X và kết tủa Y  X  đều là 1,624. Cho dung dịch Ba(OH)2 vào dung
 mY 
dịch sau khi thủy phân X9 thấy có kết tủa trắng tạo thành, không tan trong axit mạnh và có khí NH3 thoát ra.
Phần trăm về khối lượng N và S trong X9 là 14,43% và 32,99% và trong phân tử X9 chỉ chứa 1 nguyên tử lưu
huỳnh. Trong X13 có phần trăm theo khối lượng của S là 26,89% và có cầu nối chứa oxi.

Xác định các chất từ X1 đến X14 và viết các phương trình phản ứng.

37
Câu 10. (2,0 điểm) Đại cương hữu cơ (quan hệ giữa cấu trúc và tính chất)

1. Axit fumaric và axit maleic có các hằng số phân li nấc 1 (k1), nấc 2 (k2). Hãy so sánh các cặp hằng
số phân li tương ứng của hai axit này và giải thích.

2. Cho các ancol: p-CH3-C6H4-CH2OH , p-CH3O-C6H4-CH2OH, p-CN-C6H4-CH2OH và p-Cl-C6H4-


CH2OH. Hãy so sánh khả năng phản ứng của các ancol với HBr và giải thích.

------------- HẾT -------------

SỞ GD&ĐT HƯNG YÊN KỲ THI CHỌN HỌC SINH GIỎI


TRƯỜNG THPT CHUYÊN HƯNG YÊN KHU VỰC DUYÊN HẢI & ĐỒNG BẰNG BẮC BỘ
LẦN THỨ XIII
ĐỀ ĐỀ XUẤT MÔN: HÓA HỌC - LỚP 10
Thời gian làm bài: 180 phút, không kể thời gian giao đề

Câu 1. (2,0 điểm) Cấu tạo nguyên tử, phân tử, định luật tuần hoàn
1. NO được sinh ra từ động cơ đốt trong và từ quá trình phóng tia sét trong khí quyển.
a. Dựa trên thuyết MO, cho biết cấu hình electron của các MO hóa trị và bậc liên kết trong phân tử NO?
b. Trên NO có một electron chưa tham gia liên kết, electron định vị chủ yếu trên nguyên tử N hay O? Tại sao?
2.Tính năng lượng ion hoá thứ 2 của He.
Câu 2. (2,0 điểm) Tinh thể
Thời Napoleon, những chiếc cúc áo của binh sĩ được làm từ thiếc. Người ta kể rằng, khi mùa đông bắt đầu,
nhiệt độ giảm xuống dẫn đến sự thay đổi cấu trúc tinh thể của thiếc, β-Sn chuyển thành dạng α-Sn mỏng
manh hơn rất nhiều. Dưới đây là các ô mạng cơ sở của hai cấu trúc tinh thể này:

Ô mạng cơ sở α-Sn có hình lập phương, giống kim cương, với thông số a = 6,46.10-10 m. Ô mạng β-Sn
là một hình hộp chữ nhật với các thông số a = b = 5,83.10-10 m và c = 3,18.10-10 m. Cho nguyên tử khối
của Sn = 118,71
(a) Tính khối lượng riêng của các dạng thù hình của thiếc và chỉ rõ liệu những chiếc cúc áo được làm bằng
thiếc của binh lính Pháp đã bị nở ra hay co lại.
Các thông số nhiệt động học của cả hai dạng thù hình được cho dưới đây:
38
ΔfH° / kJ.mol-1 S° / J.mol-1.K-1
α-Sn -2,03 44,1
(b) Ở nhiệt độ nào β-Sn 0 51,18 thì sự chuyển hóa β-
Sn → α-Sn tự diễn biến? Giả sử rằng
enthalpy và entropy của chuyển hóa không phụ thuộc vào nhiệt độ.
Dưới đây là minh họa của một mạng tinh thể chưa rõ tỉ lệ hợp thức của thiếc oxit, SnxOy.

Mạng tinh thể SnxOy. Các nguyên tử nhỏ (xám) - Sn, lớn (đen) - O.
(c) Xác định hệ số tỉ lượng x, y trong công thức của SnxOy.
Câu 3. (2,0 điểm) Phản ứng hạt nhân
1. Hoàn thành các phản ứng hạt nhân sau:
a. 18
O + p → …+ n b. … + 12 D → 18F + 
c. 19
F + 12 D → 20F + … d. 16
O + … → 18F + p + n
2. Phản ứng (a) ở trên được dùng để tổng hợp 18F (chu kì bán hủy của 18F là 109,7 phút), nguyên liệu sử
dụng là nước được làm giàu H218O. Sự có mặt của nước thường H216O dẫn tới phản ứng phụ với 16O và hình
thành đồng vị 17F.
a. Tính hiệu suất gắn 18F vào D-glucozơ nếu hoạt độ phóng xạ ban đầu của một mẫu 18F là 600 MBq và hoạt
độ phóng xạ của 18F-2-đeoxi-D-glucozơ (FDG) sau khi gắn là 528,3 MBq. Thời gian tổng hợp là 3,5 phút.
b. Thời gian bán hủy sinh học của 18F-2-đeoxi-D-glucozơ là 120 phút. Tính hoạt độ phóng xạ còn lại theo
MBq trong bệnh nhân sau một giờ tiêm FDG? Biết hoạt độ phóng xạ ban đầu là 450 MBq.
Câu 4. (2,0 điểm) Nhiệt hóa học
1. Xét mẫu nước chứa 1 mol nước ở -30oC, sau đó đun nóng đến khi mẫu nước này ở thể hơi tại nhiệt độ
140oC. Sử dụng các dữ liệu dưới đây, tính nhiệt (q) cho toàn quá trình.
Nhiệt dung riêng của nước đá = 2,03 J/°C.g
Nhiệt dung riêng của nước = 4,18 J/°C.g
Nhiệt dung riêng của hơi nước = 2,02 J/°C.g
H2O(r) → H2O(l) Hnóng chảy = 6,02 kJ/mol (tại 0°C)
H2O(l) → H2O(k) Hbay hơi = 40,7 kJ/mol (tại 100°C)
2. Nhiệt hình thành chuẩn của H2O (l) tại 298K là -285,8 kJ/mol. Coi hệ khí là khí lý tưởng, tính biến thiên
nội năng của quá trình sau tại 298K và 1atm:
H2O (l) → H2 (k) + 1/2O2 (k) Uo = ?
Câu 5. (2,0 điểm) Cân bằng hoá học trong pha khí
Ở 27 C, phản ứng: N2O4(k)  2 NO2(k), có hằng số cân bằng Kp = 0,17 atm.
0

1. Tính thành phần phần trăm số mol của hỗn hợp khí khi áp suất chung của hệ lần lượt bằng 1 atm và 10 atm.
39
2. Tính hằng số cân bằng Kp của phản ứng trên ở 630C. Biết rằng nhiệt hình thành chuẩn ở 250C của N2O4 và
NO2 bằng 9,7 và 33,5 kJ/mol. Giả thiết rằng nhiệt của phản ứng, ∆H0, không phụ thuộc vào nhiệt độ.
3.Từ các kết quả thu được ở trên có thể rút ra kết luận gì về ảnh hưởng của áp suất và nhiệt độ đến sự dịch
chuyển cân bằng? Đồng thời liên hệ các kết quả đó với nguyên lí Lơ Sa–tơ–li–ê.
4. Cho biết chiều hướng chuyển dịch cân bằng khi:
- Giữ áp suất và nhiệt độ của hệ không đổi, thêm vào hệ một lượng khí Ar.
- Giữ thể tích và nhiệt độ của hệ không đổi, thêm vào hệ một lượng khí Ar.
Câu 6. (2,0 điểm) Động hóa học hình thức
Xét sự thủy phân của este metyl axetat ở 250C trong các môi trường sau:
Trường hợp 1: Trong môi trường kiềm, khi tăng nồng độ kiềm lên gấp đôi thì tốc độ phản ứng tăng lên hai lần.
Nhận xét này cũng được thấy khi tăng nồng độ của este lên hai lần.
Trường hợp 2: Phản ứng thuỷ phân được thực hiện trong môi trường đệm
Trường hợp 3: Người ta tiến hành phản ứng thuỷ phân trong môi trường axit HCl 0,05M dư. Động học của
phản ứng được nghiên cứu bằng cách chuẩn độ 25ml hỗn hợp phản ứng bằng dung dịch NaOH ở từng thời
điểm t với kết quả như sau :
t [phút] 0 21 75 119 
VNaOH [cm3] 24,4 25,8 29,3 31,7 47,2
1. Hãy viết phương trình động học của phản ứng. Cho biết bậc của phản ứng trong từng trường hợp.
2. Trong TH1: Nếu cho 0,01 mol xút và 0,01 mol este vào 1lit nước (xem như thể tích thay đổi không đáng
kể). Sau 200 phút thì 3/5 lượng este chưa bị phân huỷ. Tính hằng số tốc độ phản ứng k1
3. Trong TH 3: Hãy tính hằng số tốc độ phản ứng k3 và thời gian để este phân huỷ hết 50%. Từ đó hãy so sánh
giá trị k1 và k3
Câu 7. (2,0 điểm) Dung dịch và phản ứng trong dung dịch
Dung dịch X chứa H3PO4 và H2SO4 0,010M có pHX = 1,75.
1. Tính nồng độ mol/l của H3PO4 trong dung dịch X.
2. Thêm một lượng HCOOH vào dung dịch X, thu được dung dịch Y. Xác định nồng độ HCOOH có trong
dung dịch Y sao cho độ điện li của H3PO4 giảm 20% so với trước khi cho vào.
(Coi thể tích dung dịch không đổi khi thêm HCOOH).
3. Chuẩn độ dung dịch X bằng dung dịch NaOH 0,020M. Tính sai số chuẩn độ nếu dừng chuẩn độ tại pH =
4,4 (ứng với sự chuyển màu metyl da cam từ màu da cam sang màu vàng).
Cho biết: H3PO4 có pKa1 = 2,15; pKa2 = 7,21; pKa3 = 12,32;
pKa(HSO4-) = 1,99; pKa(HCOOH) = 3,75; pKw = 14,00.
Câu 8. (2,0 điểm) Phản ứng oxi hoá khử. Pin điện và điện phân
Ở 25 C, xét pin có sơ đồ: Pb | PbSO4 || CH3COOH (0,01 M) | H2 (P = 1,0 atm),Pt . Trong đó điện cực trái
o

gồm một dây Pb nhúng vào dung dịch bão hòa PbSO4.
Cho biết: E oPb2+ /Pb = −0,126 V ; E oPbSO4 /Pb = −0,359 V .
a) Tính thế của điện cực bên trái sơ đồ pin.
b) Sức điện động của pin bằng 0,0414 V. Tính pH của dung dịch axit (ở điện cực bên phải sơ đồ pin) và
hằng số axit của CH3COOH.
Câu 9. (2,0 điểm) Halogen, Oxi – lưu huỳnh

40
1. Để nhận biết ion sunfit, người ta cho vào một ống nghiệm 1 đến 2 giọt dung dịch iot, 3 đến 4 giọt
dung dịch A có chứa ion sunfit (1). Sau đó cho tiếp vào đó 2-3 giọt dung dịch HCl và vài giọt dung dịch BaCl2
thấy xuất hiện kết tủa B (2).
a) Nêu hiện tượng xảy ra trong các giai đoạn 1, 2 của thí nghiệm và viết phương trình hóa học để minh
họa.
b) Cho biết tại sao thí nghiệm nhận biết ion sunfit nêu trên thường được tiến hành trong môi trường
axit hoặc môi trường trung hòa, không được tiến hành trong môi trường bazơ?
2. Xử lí 13,16 gam hỗn hợp chất rắn X gồm hai muối khan KIOx và KIOy (y > x) bằng một lượng dư
KI trong môi trường axit thu được 200 mL dung dịch A.
a) Viết phương trình hóa học cho các phản ứng xảy ra dưới dạng ion rút gọn.
b) Lấy 25 mL dung dịch A cho vào một bình định mức 150 mL, pha loãng bằng nước cất, điều chỉnh
dung dịch về pH=3, thêm nước đến vạch. Để chuẩn độ 25 mL dung dịch trong bình định mức này cần dùng
41,67 mL dung dịch Na2S2O3 0,2M để đạt tới điểm cuối với chỉ thị hồ tinh bột. Cho biết công thức hóa học và
phần trăm khối lượng của mỗi muối trong hỗn hợp đầu biết tỉ lệ mol của chúng là 2 : 1.
Câu 10. (2,0 điểm) Đại cương hữu cơ (quan hệ giữa cấu trúc và tính chất)

1. Hãy cho biết cấu hình tuyệt đối (R/S) ở bên cạnh các nguyên tử C* trong phân tử sau:
Ph
OH
Ac-O O
NH O
O

Ph O H O
HO O O
OH Ph
O Ac

2. Có ba hợp chất: A, B và C

HO C HO C C
O CH3 CH3 CH3
O OH O
A B C
a) Hãy so sánh tính axit của A và B.
b) Hãy so sánh nhiệt độ sôi và độ tan trong dung môi không phân cực của B và C.
c) Cho biết số đồng phân lập thể có thể có của A, B và C.

-------- HẾT --------


TRƯỜNG THPT CHUYÊN KÌ THI CHỌN HỌC SINH GIỎI KHU VỰC DUYÊN HẢI VÀ
NGUYỄN TRÃI ĐỒNG BẰNG BẮC BỘ
TỈNH HẢI DƯƠNG ĐỀ GIỚI THIỆU MÔN HÓA HỌC LỚP 10
Thời gian làm bài: 180 phút
NĂM 2022
Câu 1. (2,0 điểm) Cấu tạo nguyên tử, phân tử, định luật tuần hoàn

41
1. a) Viết công thức Lewis cho PF5. Sử dụng mô hình VSEPR để biểu diễn cấu trúc cho biết dạng hình học của phân
tử này.
b) Ở trạng thái rắn Photphopentaclorua có cấu trúc ion với sự có mặt hai ion PCl +4 và PCl6− . Sử dụng mô

hình VSEPR hãy biểu diễn và cho biết dạng hình học của các ion này.
2. a) Vẽ giản đồ năng lượng obitan phân tử (MO) cho phân tử O2. Từ đó cho biết cấu hình electron của các tiểu

phân: O2; O2 ; O+2 và O22+

b) Gán các giá trị độ dài liên kết sau cho các liên kết trong mỗi tiểu phân: 1,49 Å ; 1,26 Å ; 1,21 Å và 1,12 Å.

c) Sắp xếp năng lượng ion hóa của các tiểu phân sau đây theo thứ tự từ thấp đến cao : O, O2, O2 ; O+2 . Giải
thích.
Câu 2. (2,0 điểm) Tinh thể
Bằng phương pháp nhiễu xạ tia X, người ta đã ghi được các kết quả sau:

- Ở 20oC, NH4Cl kết tinh theo mạng lập phương với hằng số mạng a = 3,88 Å và khối lượng riêng d = 1,5
g/cm3.

- Ở 250oC, NH4Cl kết tinh theo mạng lập phương với hằng số mạng a = 6,53 Å và khối lượng riêng d = 1,3
g/cm3.

Từ các dữ kiện trên hãy cho biết:

1. Số phân tử NH4Cl có trong một ô mạng cơ sở, từ đó kết luận về kiểu mạng của các tinh thể hình thành
ở 20oC và 250oC.

2. Khoảng cách N – Cl theo Å cho từng kiểu mạng tinh thể đã xác định ở (a).

Câu 3. (2,0 điểm) Phản ứng hạt nhân

1. Ban đầu có một mẫu phóng xạ X nguyên chất. Tại thời điểm t1 (s) thì có 80% mẫu phóng xạ đã bị phân rã.
Đến thời điểm t2 = t1 + 100 (s) thì số hạt nhân X chưa bị phân rã còn lại 5% so với ban đầu. Tính chu kì bán
hủy của hạt nhân nguyên tử X? Thời gian t1 và t2?
2. Tính năng lượng giải phóng (đơn vị J) ứng với 1 nguyên tử và 1 mol nguyên tử 92U235 theo phản ứng sau:
92U
235
+ 0n1 → 47La146 + 35Br87 + ?

Biết khối lượng của 92U235; 0n1; 57La146; 35Br87 lần lượt là 235,044u; 1,00861u; 145,943u; 86,912u. Năng
lượng ứng với 1u là 931,2 MeV và 1eV = 1,602.10−19J; NA = 6,02.1023.

Câu 4. (2,0 điểm) Nhiệt hóa học


1. Cho các dữ kiện sau:
Năng lượng kJ.mol¯1 Năng lượng kJ.mol¯1
thăng hoa của Na 108,68 liên kết của Cl2 242,60
ion hóa thứ nhất của Na 495,80 mạng lưới NaF 922,88
liên kết của F2 155,00 mạng lưới NaCl 767,00
42
Nhiệt hình thành của NaF rắn : -573,60 kJ.mol-1
Nhiệt hình thành của NaCl rắn: -401,28 kJ.mol-1
Tính ái lực electron của F và Cl ; so sánh các kết quả thu được và giải thích.
2. Biết giá trị nhiệt động của các chất sau ở điều kiện chuẩn là:
Chất Fe O2 FeO Fe2O3 Fe3O4

ΔH0s(kcal.mol-1 0 0 -63,7 -169,5 -266,9


S0 (cal.mol-1.K-1) 6,5 49,0 14,0 20,9 36,2
a) Tính biến thiên năng lượng tự do Gibbs ( ΔG0) của sự tạo thành các oxit sắt từ các đơn chất ở điều kiện
chuẩn.
b) Cho biết ở điều kiện chuẩn oxit sắt nào bền nhất ?
Câu 5. (2,0 điểm) Cân bằng hoá học trong pha khí
⎯⎯
→ 2SO3(k) (*) được thiết lập ở 450 K người ta xác định được
Trong một hệ có cân bằng 2SO2(k) + O2(k) ⎯

các áp suất riêng phần sau đây:
P02 = 0,124.105 Pa; PSO2 = 0,375.105 Pa; PSO3 = 0,501.105 Pa

1. Tính hằng số cân bằng Kp và ΔG0 của phản ứng (*) ở 450 K.
2. Tính lượng O2 và SO3, biết hệ có 500 mol SO2.
3. Thêm 10 mol SO2 vào hệ này đồng thời giữ cho nhiệt độ và áp suất tổng cộng không đổi. Bằng cách tính,
hãy cho biết cân bằng (*) chuyển dịch theo chiều nào?
4. Trong một hệ cân bằng SO2/O2/SO3 ở 460 K và áp suất tổng cộng 1.105 Pa, người ta tìm được: Kp = 1,250.10-
5
Pa-1, n(SO2) = 450 mol , n(O2) = 100 mol và n (SO3) = 300 mol. Nếu thêm 10 mol O2 vào hệ này đồng thời
giữ cho nhiệt độ và áp suất không đổi thì cân bằng chuyển dịch theo chiều nào?
Cho: Áp suất tiêu chuẩn P0 = 1,013.105 Pa; R = 8,314 JK-1mol-1; 1atm = 1,013.105 Pa.
Câu 6. (2,0 điểm) Động hóa học hình thức
Trong dung dịch nước, chất A bị phân hủy phương trình:
A + 2H2O → 2X+ + Y2- (1)
Trong dung dịch loãng, hằng số tốc độ của phản ứng tại 350 K là k1 = 4,00.10-5 s-1.
1. Cho biết bậc của phản ứng (1).
2. Tính thời gian cần thiết, t1, để 80% lượng A bị phân hủy ở 350K.
3. Tính hằng số tốc độ của phản ứng, k2, tại 300K và thời gian cần thiết, t2, để 80% lượng A bị phân hủy ở
nhiệt độ này. Biết năng lượng hoạt hóa của phản ứng là 166,00 kJ.mol-1 và không phụ thuộc vào nhiệt độ.
4. Khi có mặt chất xúc tác, hằng số tốc độ của phản ứng phân hủy tại 300 K là k2’ = 3,00.104 s-1. Giả sử
thừa số tần số không thay đổi, tính năng lượng hoạt hóa của phản ứng khi có mặt xúc tác.
Câu 7. (2,0 điểm) Dung dịch và phản ứng trong dung dịch

43
Hấp thụ hoàn toàn 0,010 mol khí H2S vào nước cất, thu được 100,0 mL dung dịch A.
1. Tính nồng độ cân bằng của các ion trong dung dịch A.
2. Trộn 10,0 mL dung dịch A với 10,0 mL dung dịch FeCl2 0,02 M, thu được 20,0 mL dung dịch B.
Có kết tủa xuất hiện từ dung dịch B hay không?
3. Tính giá trị pH của dung dịch B để có thể tách được ion Fe2+ hoàn toàn ra khỏi dung dịch dưới
dạng kết tủa, biết rằng ion Fe2+ được coi là tách hoàn toàn ra khỏi dung dịch khi nồng độ còn lại của sắt(II)
trong dung dịch là 10–6 M.
4. Để điều chỉnh pH của dung dịch B đến khi kết tủa hoàn toàn ion Fe2+ (nồng độ còn lại của sắt(II)
trong dung dịch là 10–6 M) ta có thể dùng dung dịch đệm axetat. Tiến hành như sau, đầu tiên cho CH3COOH
đặc vào 20,0 mL dung dịch B đến nồng độ 0,10 M; sau đó cho từ từ CH3COONa vào dung dịch thu được đến
khi hết tủa hoàn toàn Fe2+ thì hết m (gam). Tính giá trị của m. Coi thể tích dung dịch không đổi sau khi cho
thêm đệm axetat.
Cho biết: pKS(FeS) = 17,2; pKa1(H2S) = 7,02; pKa2(H2S) = 12,90;
pKa(CH3COOH) = 4,75; *(FeOH+) = 10-5,92; M(CH3COONa) = 82.
Câu 8. (2,0 điểm) Phản ứng oxi hoá khử. Pin điện và điện phân
1. Dưới đây là giản đồ Latimer của một chuỗi tiểu phân chứa lưu huỳnh ở pH = 0. Các giá trị thế tính theo
Volt:

a. Xác định các giá trị x, y còn thiếu.


b. Cho biết S (0) bền hay không bền.
c. Viết phương trình tự oxi hóa khử của S2O32- dựa theo các tiểu phân được cho trong giản đồ Latimer.
d. Tính ∆G0 và từ đó tính hằng số cân bằng của phản ứng tự oxi hóa – khử ở 25oC.
Cho : F = 96500 ; ln = 2,303lg; R = 8,314(J/K)
2. a. Hãy trình bày cách thiết lập sơ đồ pin sao cho khi pin hoạt động thì xảy ra phản ứng:

H3AsO4 + NH3 → H 2 AsO −4 + NH +4

b. Tính sức điện động của pin ở điều kiện tiêu chuẩn ( E pin ).

Cho:
pK ai(H3AsO4 ) = 2,13; 6,94; 11,50; pK a(NH+ ) = 9, 24 (pKa = - lgKa, với Ka là hằng số phân li axit).
4

RT
p H 2 = 1 atm; ở 25 oC: 2,303 = 0, 0592.
F
44
Câu 9. (2,0 điểm) Halogen, Oxi – lưu huỳnh
1) Vì sao trong phân tử Cl2, Br2, I2 ngoài liên kết  còn có liên kết  ? Vì sao độ bền nhiệt tăng từ F2 đến Cl2,
sau đó lại giảm từ Cl2 đến I2?
2) Trong số các cacbonyl halogenua COX2 người ta chỉ điều chế được 3 chất: cacbonyl florua COF2, cacbonyl
clorua COCl2 và cacbonyl bromua COBr2. Vì sao không có hợp chất cacbonyl iođua COI2
3) Giải thích tại sao các ion XO- bền hơn các axít HXO tương ứng và độ bền giảm từ ClO- đến IO-.
4) Giải thích các hiện tượng sau: SnS2 tan trong (NH4)2S; SnS không tan trong dung dịch (NH4)2S nhưng tan
trong dung dịch (NH4)2S2.
Câu 10. (2,0 điểm) Đại cương hữu cơ (quan hệ giữa cấu trúc và tính chất)
1. Xác định cấu dạng bền của những hợp chất sau:
Cl

O HO OH
Me
B
A
Trong dung môi: a) metanol; b) octan
2. Tính axit – bazơ
a) Giải thích giá trị pKa tăng dần

b) Cho biết chất nào có tính base mạnh nhất trong số các chất sau đây. Giải thích.

--------------------
KỲ THI HỌC SINH GIỎI CÁC TRƯỜNG THPT CHUYÊN
KHU VỰC DUYÊN HẢI VÀ ĐỒNG BẰNG BẮC BỘ
LẦN THỨ XIV, NĂM 2022
ĐỀ THI MÔN: HÓA HỌC 10
Thời gian: 180 phút (không kể thời gian giao đề)
ĐỀ THI ĐỀ
XUẤT
45
(Đề thi gồm 5 trang)

Câu 1. (2,0 điểm) Cấu tạo nguyên tử, phân tử, định luật tuần hoàn
1. Sử dụng mô hình VSEPR dự đoán dạng hình học của các ion và phân tử sau: XeF4, BCl3, NF3, S2O 32 − , SiF62-

, NO +2 , I 3− , IF5.
2. Giải thích:
a) Nhiệt độ sôi của NH3 (-33oC) cao hơn nhiệt độ sôi của NF3 (-129oC) nhưng thấp hơn của NCl3 (71oC).
b) Sự biến đổi góc liên kết: NH3 107o → PH3 93,6o
PH3 93,6o → PF3 96,3o
Câu 2.(2,0 điểm) Tinh thể
Vanadi (V) có khối lượng riêng là 5,96 g/cm3, kết tinh theo cấu trúc mạng lập phương với cạnh của ô
mạng cơ sở là 307 pm. Biết khối lượng mol nguyên tử của Vanadi là 50,94.
1. Vanadi kết tinh theo kiểu mạng lập phương nào ?
2. Số phối trí của Vanadi trong cấu trúc này là bao nhiêu? Giải thích.
3. Tính phần trăm thể tích không gian trống trong ô mạng cơ sở của Vanadi?
Câu 3. (2,0 điểm) Phản ứng hạt nhân
1. Cacbon tự nhiên chứa 2 đồng vị bền là 12C (98,9% khối lượng) và 13C (1,1% khối lượng) cùng lượng vết
đồng vị phóng xạ 14C (phân rã β–, t1/2 = 5730 năm). Hoạt độ phóng xạ riêng của cacbon trong cơ thể sống là

230Bq.kg–1. Năm 1983, người ta tìm thấy một con thuyền cổ chìm ngoài khơi Đại Tây Dương. Cacbon trong
gỗ của con thuyền này có hoạt độ phóng xạ riêng là 180Bq.kg–1.
a) Tỉ lệ số nguyên tử giữa các đồng vị 13C/12C và 14C/12C trong cơ thể sống là bao nhiêu?
b) Cây để dùng làm gỗ đóng thuyền trên được đốn hạ vào năm nào?
2. Trong y học hạt nhân, các đồng vị phóng xạ được bắn phá vào nơi có sự phân chia tế bào để tiêu diệt chúng.
Một trong các kĩ thuật đó là xác định độ phóng xạ khi pha loãng trong máu của bệnh nhân. Ba hợp chất dược
phẩm phóng xạ lần lượt có chứa các đồng vị phóng xạ 71Zn (t1/2 = 2,4 phút), 67Ga (t1/2 = 78,25 giờ) và 68Ge (t1/2
= 287 ngày).
a) Kiểu phóng xạ của ba đồng vị này là bức xạ β (71Zn) và bắt electron (67Ga và 68Ge). Sản phẩm của quá trình
phóng xạ này là gì?
b) Không kể đến hiệu ứng hóa học, 67Ga có thuận lợi gì hơn hai đồng vị kia trong việc điều trị?
Câu 4 (2 điểm). Nhiệt hóa học
Khi đốt cháy 3,9 g hơi benzen ở 250C, 1atm với một lượng oxi dư toả ra 163400 J sản phẩm là CO2(k) và
H2O(l).
1. Hãy tính nhiệt toả ra khi đốt cháy 7,8g hơi benzen và oxi dư trong bom nhiệt lượng kế ở 250C sản phẩm là
CO2(k) và H2O(l).
46
2. Tính nhiệt độ của ngọn lửa benzen cháy ở áp suất 1,0 atm, 250C trong 2 trường hợp sau đây:
a. Trong không khí (20% oxi và 80% nito về thể tích).
b. Trong oxi nguyên chất .
C0p (CO2 khí) = 26,80 +42,3.10-3T (J/mol.K)
C0p (N2 khí) = 27,10 +6,00.10-3T (J/mol.K)
∆H0 bay hơi của nước lỏng ở 373K là 40,66kJ/mol;
C0p của nước lỏng là 75,3 J/mol.K;
C0p của nước khí là 30,2+1,00.10-2T (J/mol.K);
Câu 5 (2 điểm). Cân bằng hoá học trong pha khí
Xét cân bằng của phản ứng sau: N2O4 (k) ⇄ 2NO2 (k) tại nhiệt độ T và áp suất P. Cho biết dữ kiện nhiệt động
học sau:
ΔH0ht, 298 (kJ/mol) S0298 (J.K-1.mol-1)
N2O4 9,37 304,3
NO2 33,89 240,45
a. Ở điều kiện chuẩn, N2O4 có tự phân li không?
b. Giả thiết trong khoảng nhiệt độ 298K → 318K, nhiệt phản ứng không phụ thuộc nhiệt độ. Hãy tính
hằng số cân bằng Kp tại 298K và 318K
c. Tại p=2atm, tính độ phân li α tại các nhiệt độ 298K và 318K? Nhận xét kết quả thu được.
Câu 6 (2 điểm). Động hóa học hình thức
Ở 690K, etylen oxit bị nhiệt phân theo phản ứng:

Trong đó chất đầu và sản phẩm đều nằm ở pha khí.


Để khảo sát động học của phản ứng này, người ta đo áp suất tổng cộng của hệ phản ứng trong bình kín theo
thời gian, dữ liệu thu được như sau:
t (phút) 10 20 40 60 100 200 ∞
Ptổng (mmHg) 139,14 151,67 172,65 189,15 212,34 238,66 249,88
Xác định bậc và hằng số tốc độ của phản ứng trên. Tính thời gian bán phản ứng (giả sử ban đầu chỉ có 1 chất
trong bình phản ứng).
Câu 7 (2 điểm). Dung dịch và phản ứng trong dung dịch
Trộn 10,00 mL dung dịch CH3COOH 0,20 M với 10,00 mL dung dịch H3PO4, thu được dung dịch A có pH =
1,50.
a. Tính CH3PO4 trong dung dịch H3PO4 trước khi trộn.

47
b. Tính độ điện li của CH3COOH trong dung dịch A.
c. Thêm từ từ Na2CO3 rắn vào dung dịch A cho đến pH = 4,0, thu được dung dịch B. Tính số gam
Na2CO3 đã dùng.
Cho biết: H3PO4: pKa1 = 2,15; pKa2 = 7,21; pKa3 = 12,32;
CH3COOH: pKa = 4,76; CO2 + H2O có pKa1 = 6,35; pKa2 = 10,33;
Câu 8 (2 điểm). Phản ứng oxi hoá khử. Pin điện và điện phân
Một pin điện tạo bởi: Một điện cực gồm tấm Cu nhúng trong dung dịch CuSO4 0,5 M, điện cực thứ hai là
một dây Pt nhúng trong dung dịch Fe2+, Fe3+ với lượng [Fe3+] = 2[Fe2+] và một dây dẫn nối Cu với Pt.
a. Viết sơ đồ pin, phản ứng điện cực và tính sức điện động ban đầu của pin.
[Fe3+ ]
b. Cho rằng thể tích dung dịch CuSO4 khá lớn, xác định tỷ số khi pin ngừng hoạt động.
[Fe2+ ]
c. Trộn ba dung dịch: 25 ml Fe(NO3)2 0,1 M ; 25 ml Fe(NO3)3 1,0 M ; 50 ml AgNO3 0,6 M và thêm
[Fe3+ ]
một số mảnh Ag vụn. Xác định chiều phản ứng và tính giá trị tối thiểu của tỷ số để phản ứng đổi chiều?
[Fe2+ ]
Cho: Eo (Cu2+/Cu) = 0,34 V ; Eo (Fe3+/Fe2+) = 0,77 V ; Eo(Ag+/Ag) = 0,8 V.
Câu 9 (2 điểm). Halogen, Oxi – lưu huỳnh
1. Nguyên tố X là một phi kim. Hợp chất khí của X với hiđro là E; oxit cao nhất của X là F. Tỉ khối hơi của F
so với E là 5,0137.
a. Tìm X.
b. Hoàn thành sơ đồ sau (biết X3, X4, X6 là muối có oxi của X; X5 là muối không chứa oxi của X; X7 là axit
không bền của X).

(12) (11)
X7 X6 X5

(6) X1
(10)
(8) (7) (1) (2)
(5) (3)
X4
X3 X X2
(9) (4) + Fe

2. Để khảo sát sự phụ thuộc thành phần hơi của B theo nhiệt độ, người ta tiến hành thí nghiệm sau đây:
Lấy 3,2 gam đơn chất B cho vào một bình kín không có không khí, dung tích 1 lít. Đun nóng bình để
B hoá hơi hoàn toàn. Kết quả đo nhiệt độ và áp suất bình được ghi lại trong bảng sau:
Nhiệt độ (oC) Áp suất (atm)
444,6 0,73554
450 0,88929
500 1,26772

48
900 4,80930
1500 14,53860
Xác định thành phần định tính hơi đơn chất B tại các nhiệt độ trên và giải thích.
Cho: R = 0,082 L.atm.K-1.mol-1
Câu 10. (2,0 điểm) Đại cương hữu cơ (quan hệ giữa cấu trúc và tính chất)
1. Cho các giá trị pKa: 5,06; 7,79; 10,58 và các chất hữu cơ sau:

Hãy gán giá trị pKa đúng cho mỗi chất trên.
2. Dự đoán tính tan trong nước của các phân tử hữu cơ sau:

-------------- HẾT --------------


SỞ GD-ĐT TỈNH VĨNH PHÚC KỲ THI HSG CÁC TRƯỜNG THPT CHUYÊN KHU
TRƯỜNG THPT CHUYÊN VỰC DUYÊN HẢI VÀ ĐỒNG BẰNG BẮC BỘ
VĨNH PHÚC NĂM HỌC 2021 – 2022
Môn thi: Hóa học - Lớp 10
ĐỀ ĐỀ NGHỊ

Thời gian: 180 phút (không kể thời gian giao đề)


Cho: H= 1; C= 12; N = 14; O = 16; S = 32; Cl = 35,5; Ca = 40; Cu = 63,5; ZN = 7; ZO = 8; ZF = 9; ZCu = 29;
RT 0,0592
e = 1,6.10-19 C; me = 9,1.10-31kg; C = 3.108m.s-1; Số Avogadro NA = 6,023.1023 mol-1; ln = lg (ở
nF n
25oC); R =8,314 J.mol-1.K-1=0,082 l.atm. mol-1K-1;

F=96485C.mol-1; ɛ0=8,854.10-12C2J-1m-1; h = 6,626.10-34J.s; 1 bar = 105Pa; bar = 0,9869 atm; 1 atm = 101325
Pa

49
Câu 1. (2,0 điểm) Cấu tạo nguyên tử, phân tử, định luật tuần hoàn
1. Năng lượng En (J) của electron trong X (là hạt một electron) được tính bằng biểu thức:
𝑚𝑒 . 𝑍 2 𝑒 4
𝐸𝑛 = − 2 2 2
8𝜀0 𝑛 ℎ
Trong đó, e là điện tích nguyên tố; Z là điện tích hạt nhân;  o là hằng số điện; h là hằng số Planck; n là số lượng
tử chính (n = 1, 2, 3…).
a) Nếu X là nguyên tử hidro. Tính bước sóng λmax (nm) của dãy phổ Lyman khi electron chuyển từ n =
3 về n = 1.
b) Sự chuyển electron từ các trạng thái năng lượng cao hơn về năng lượng E3 (n=3) chỉ cho một vạch
ở vùng nhìn thấy trong số các tín hiệu quang phổ của X. Xác định Z và bước sóng (theo nm) của vạch phổ đó.
Biết ánh sáng nhìn thấy có bước sóng trong khoảng từ 380 nm đến 800 nm.
2. Cho các phân tử và ion sau: N2, O2, N22-, N2, O2+
a) Viết cấu hình electron của các phân tử và ion ở trên
b) Sắp xếp các tiểu phân (phân tử hay ion) này theo thứ tự tăng dần năng lượng ion hóa thứ nhất. Giải thích.
Câu 2. (2,0 điểm) Tinh thể
1. Nhôm oxit bị nén dưới áp suất cao tạo thành một dạng đa hình của nó, có cấu trúc hình thoi, với các thông
số của ô mạng cơ sở a= 0,6393 nm, b = 0,4362 nm và c = 0,4543 nm, và các góc α = β = γ = 90o. Tính khối
lượng riêng của dạng đa hình này của nhôm oxit theo g/cm3, biết rằng mỗi ô mạng cơ sở chứa 4 đơn vị phân
tử nhôm oxit.
2. Ô mạng cơ sở của palladium có dạng lập phương tâm diện. Tâm của cả các nguyên tử palldium đặt ở các
đỉnh và các tâm mặt của hình lập phương (các nguyên tử được xem như những quả cầu cứng). Khối lượng
riêng của palladium là 12.02 gam/cm3, khối lượng mol là 106.4 gam/mol.
a) Tính số nguyên tử palladium trong 1 ô mạng cơ sở, độ dài cạnh (a) và bán kính của các nguyên tử
palladium (r) theo pm (1 pm = 10-12 m).
b) Giả sử rằng các nguyên tử là những quả cầu cứng, tính bán kính nguyên tử cực đại có thể vừa khít
với các khoảng trống (hốc) giữa các nguyên tử palladium trong mạng tinh thể.
c) Khoảng cách liên nhân trong các phân tử H2 là 74 pm và bán kính của nguyên tử hidro là 54 pm. Hãy tính
số nguyên tử hidro cực đại có thể lấp vào ô mạng cơ sở palladium.
Câu 3. (2,0 điểm) Phản ứng hạt nhân
1. Phản ứng hạt nhân
18
8 X + 11 H → 01 n + 189Y dùng điều chế 189Y , vật liệu bia sử dụng là nước thông thường
16
đã được làm giàu bằng H2X18. Sự có mặt của nước thường H2X16 xảy ra phản ứng phụ từ 8 X tạo ra 179Y .
18
Y
Cứ 5 phút, sau khi hoàn thành phản ứng hạt nhân với vật liệu bia ở trên thì tỉ số hoạt độ phóng xạ 9
17
= 105
9Y
. Tính hàm lượng của H2X18 trong vật liệu bia, cho biết chu kì bán hủy của 18Y và 17Y lần lượt là 109,7
H 18 X →18 Y
phút và 65 giây; tỉ lệ hiệu suất = 144,7 .
H 16 X →17 Y
2. Trong sự phân hạch do dùng nơtron bắn phá 235
92 U , cuối cùng thường thu được hai sản phẩm bển là 98
42 Mo
136
và 54 Xe .
a) Những hạt cơ bản nào được phát ra.
50
b) Tính năng lượng được giải phóng ra trong mỗi phân hạch theo MeV và theo Joules (Jun)

c) Tính năng lượng được giải phóng khi mỗi gam 235
92 U bị phân hạch theo đơn vị kW-giờ. Cho:
1
235
U 136
Xe 98
Mo 0 n

Nguyên tử khối 235,04393 135,90722 97,90551 1,00867

Câu 4. (2,0 điểm) Nhiệt hóa học


Cho các phản ứng:
4Cu(r) + O2(k) 2Cu2O(r) (1)
2Cu(r) + O2(k) 2CuO(r) (2)
Cu2O(r) CuO(r) + Cu(r) (3)
2H2(k) + O2(k) 2H2O(k) (4)
1. Biết tại 300K và tại 1300K, G0 của phản ứng (1) lần lượt là -300kJ.mol-1 và
-160kJ.mol-1; G0 của phản ứng (2) lần lượt là -260 kJ.mol-1 và -80 kJ.mol-1. Tính H0 và S0 của phản
ứng (1) và (2). Biết H0 và S0 không phụ thuộc vào nhiệt độ.
2. Lập biểu thức sự phụ thuộc của G0(3) vào nhiệt độ đối với phản ứng (3)
3. Trên cơ sở tính toán, hãy dự đoán sản phẩm thu được khi nung bột Cu ở 800K trong không khí (20%
O2, 80% N2 về thể tích) ở áp suất 1 bar.
4. Trong khoảng nhiệt độ 300 – 1500 K, Cu, CuO và Cu2O có thể tồn tại trong cùng một hỗn hợp không.
Giải thích bằng tính toán?
5. Cho từ từ H2 vào bình có chứa một lượng bột CuO ở 1150 K thì CuO sẽ bị khử thành Cu. Biết phản ứng
(4) có G0(4) = - 490 + 0,1T (kJ.mol-1). Tính toán và lập luận để chỉ ra rằng H2 sẽ khử hết CuO thành
Cu2O rồi mới khử tiếp Cu2O thành Cu.
Câu 5. (2,0 điểm) Cân bằng hoá học trong pha khí
1. Khí màu nâu NO2 được giữ trong một thiết bị có thể tích thay đổi nhờ một pittông. Kết quả thí nghiệm
nén khí từ 1000ml xuống 100ml tại 25oC cho thấy có sự tồn tại cân bằng:
2NO2(k) ⇌ N2O4(k) Kp = 7,07.
Quá trình thí nghiệm thu được các kết quả sau:

V (mL) 10,0 5,0 2,0 1,0 0,5 0,2 0,1

Ptổng (atm) 0,128 0,331 0,765 1,215 1,215 1,215 a

a) Hãy cho biết giá trị a. Giải thích ?


b) Cho biết các quá trình hóa học xảy ra và tính áp suất riêng phần của mỗi tiểu phân trong trường hợp
V = 1,0 mL. Tính hằng số cân bằng của quá trình để giải thích hiện tượng “trần áp suất” tại 1,215 atm.
2. Cho phản ứng CO tác dụng với hơi nước ở 4500C theo phản ứng:
H2O(h) + CO(k) ⇌ H2(k) + CO2(k) (a)
Cho 2 mol H2O và 1 mol CO vào bình chân không ở 450oC. Khi phản ứng (a) đạt đến cân bằng hỗn hợp phản
ứng chứa 0,9 mol CO2.

51
a) Tính hằng số cân bằng của phản ứng (a) ở 450oC.
b) Phản ứng (a) cần được tiến hành ở nhiệt độ bao nhiêu để 99% lượng CO ban đầu bị chuyển hóa thành
CO2.
Cho biết: Biến thiên entanpy tạo thành (kJ.mol-1) của các chất ở 25oC và 1 atm:
CO(k): -110,5; CO2(k): -393,51; H2O(h): -241,84
Câu 6. (2,0 điểm) Động hóa học hình thức ( Không có phần câu hỏi cơ chế động học )
1. Nitrogen oxide phân hủy thành nitrogen và oxygen ở nhiệt độ 5650C, phản ứng tỏa nhiệt.
2N2O(k) → 2N2(k) + O2(k)
Phản ứng này tuân theo quy luật động học bậc 2 khi thực hiện hoàn toàn trong pha khí.
a) Nồng độ đầu của N2O là 0,108 mol.l-1, hằng số tốc độ phân hủy bậc hai của N2O là
1.10-3l.mol-1.s-1. Tính nồng độ N2O sau 1250 giây ở 5650C.
b) Năng lượng hoạt hóa của phản ứng bậc hai ở 5650C là 234 kJ.mol-1. Tính hằng số tốc độ phản ứng
ở 6000C?
2. Xét các phản ứng song song

k1
Các năng lượng hoạt hóa ứng với k1 và k2 lần lượt là 45,3 và 69,8 kJ.mol-1. Biết ở 320K thì = 1, 00 .
k2
k1
Xác định nhiệt độ mà tại đó = 2, 00 .
k2

3. Nghiên cứu về động học phản ứng Michael của β-nitrostyrene với piperidine và trạng thái chuyển tiếp của
nó. Sơ đồ phản ứng như sau:

Phương trình tốc độ của phản ứng là:


k
v = 1 .k2 .[  − nitrostyrene].[ piperidine]2
k−1
a) Xác định bậc toàn phần của phản ứng và bậc riêng phần đối với β-nitrostyrene và piperidine.
b) Biết rằng giai đoạn đầu của phản ứng là một cân bằng thuận nghịch, giai đoạn thứ 2 được xúc tác
bởi phân tử piperidine và là giai đoạn quyết định tốc độ phản ứng. Cả hai giai đoạn đều là phản ứng
cơ bản. Thiết lập phương trình tốc độ tạo thành sản phẩm. Tính hằng số tốc độ khả kiến.
Câu 7. (2,0 điểm) Dung dịch và phản ứng trong dung dịch
Dung dịch X chứa 2 đơn axit yếu là HA (hằng số phân li axit HA là pKa1 = 6,76) và HB (hằng số phân
li axit HB là pKa2 = 6,87) có pH = 3,75.
1. Chuẩn độ 10,0 mL dung dịch X cần 10,0 mL dung dịch NaOH 0,220 M đến điểm tương đương.
a) Tính nồng độ ban đầu của mỗi axit trong dung dịch X.
52
b) Tính pHtđ tại điểm tương đương.
2. Một dung dịch đệm được thêm vào dung dịch X để giữ pH của dung dịch bằng 10,0 thu được dung dịch Z.
Coi như thể tích dung dịch không đổi khi thêm dung dịch đệm vào.
Tính độ tan (theo đơn vị mol·L–1) của M(OH)2 trong dung dịch Z, biết rằng các anion A– và B– có thể
tạo phức với ion M2+.
Cho biết: pKs(M(OH)2) = 11,51.

Phức của ion M2+ và anion A– có: 1 = 2,1.103; 2 = 1,05.106.

Phức của ion M2+ và anion B– có: 1’ = 6,2.103; 2’ = 2,05.106.
Câu 8. (2,0 điểm) Phản ứng oxi hoá khử. Pin điện và điện phân
1. Pin điện hóa sau đây dựa trên phản ứng ở pha rắn và hoạt động thuận nghịch ở 1000K dưới dòng khí O2.
Các ion F- khuếch tán thông qua CaF2(r) ở 1000K.
(-) MgF2(r), MgO(r) | CaF2(r) | MgF2(r), MgAl2O4(r), Al2O3(r) (+)
Phản ứng tổng xảy ra trong pin như sau: Al2O3(r) + MgO(r) ⇌ MgAl2O4(r)
a) Viết phương trình phản ứng ở mỗi điện cực và phương trình Nernst cho mỗi nửa pin khi pin hoạt động.
Coi áp suất O2(k) là như nhau ở hai điện cực. Nồng độ của ion F- là bằng nhau ở hai điện cực và được
duy trì bởi dòng khuếch tán ion F- thông qua CaF2(r).
b) Tính ∆Go của phản ứng (ở 1000K) biết rằng, Eo (ở 1000K) của phản ứng là 0,1529V.
c) Sức điện động chuẩn của pin trong khoảng nhiệt độ từ 900K đến 1250 K là:
Eo(V) = 0,1223 + 3,06.10-5T. Tính các giá trị ∆Ho và ∆So coi những đại lượng này không phụ thuộc vào nhiệt
độ.
2. Giá trị Eo cho các bán phản ứng của các ion sắt và ion xeri (Ce) như sau:

Fe3+ + e → Fe2+ Eo = +0,77V

Ce4+ + e → Ce3+ Eo = +1,61V


Hai chất chỉ thị sau được dùng để xác định điểm tương đương trong các phép chuẩn độ oxy hóa – khử:

Di-bolane (Dip): Inox + 2e → Inkh ; Eodip = +0,76V


(Tím) (Không màu)

p-nitro-di-bolane(pn): Inox + 2e → Inkh ; Eodip = +1,01V


(Tím) (Không màu)
Cả hai chất chỉ thị trên đều đổi màu khi tỉ lệ nồng độ [Inox]/[Inkh] = 10. Bằng tính toán, cho biết chất chỉ thị nào
ở trên thích hợp cho phép chuẩn độ Fe3+ bằng Ce4+.
Câu 9. (2,0 điểm) Halogen
Các hợp chất An là các oxit của nguyên tố clo tạo thành theo sơ đồ sau:

53
A3

3) AgZ A4 6) O3 A5
5) H2C2O4
2) HgO
4) KOH
Hiệu phần trăm
A2 khối lượng KZtố clo và oxi cùng một số tính chất vật lí của các hợp chất này được
X các nguyên
500C 900C
cho trong bảng sau đây: 0
1) AgY 7) 400 C 8) H2SO4 9)P2O5
KY HY A6
Chất A1 A2 A3 t 0C
A4 A5
t 0C
A6

m1 (%) − m2 (%)
A1 5.2 63.2 19.3 5.2 -15.0 -22.4

chất
chất khí màu không khí màu chất
Tính chất lỏng
lỏng vàng bền vàng lỏng
nâu đỏ
1. a) Xác định các chất A1 - A6
b) Vẽ công thức cấu tạo các chất A1 - A6 .
2. Viết tất cả các phương trình phản ứng đã trình bày trong sơ đồ.
Câu 10. (2,0 điểm) đại cương vô cơ
1. Cho các hợp chất sau:

a) So sánh lực acid của cyclohexane-1,4-dione, cyclohexane-1,3-dione và bicyclo[2.2.2]octane-2,6-


dione (BOD). Giải thích ngắn gọn.
b) Giải thích vì sao:
- pKa của trypticene lớn hơn nhiều so với pKa của triphenylmethane.
- Tuy nhiên, pKa of fluoradene lại nhỏ hơn nhiều so với pKa của triphenylmethane.
2. Hãy cho biết hợp chất dưới đây có bao nhiêu đồng phân lập thể và biểu diễn cấu trúc không gian của các
đồng phân lập thể đó.

----------------------------

54
KỲ THI HỌC SINH GIỎI CÁC TRƯỜNG THPT CHUYÊN
KHU VỰC DUYÊN HẢI VÀ ĐỒNG BẰNG BẮC BỘ
LẦN THỨ XIII, NĂM 2022

ĐỀ THI MÔN: HÓA HỌC-KHỐI 10


Thời gian: 180 phút

ĐỀ GIỚI
THIỆU
CÂU 1.
1.1. Có cấu hình electron 1s22s22p63s23p63d54s1 (1)
a) Dùng kí hiệu ô lượng tử biểu diễn cấu hình electron (1).
b) Cấu hình electron (1) là cấu hình electron của nguyên tố hay ion ? Tại sao ?
c) Cho biết tính chất hoá học đặc trưng của ion hay nguyên tố ứng với cấu hình electron (1), hãy viết
một phương trình phản ứng để minh họa.
Z2
1.2. Biết En = -13,6. 2 (n: số lượng tử chính, Z: số đơn vị điện tích hạt nhân).
n
a) Tính năng lượng 1e trong trường lực một hạt nhân của mỗi hệ N6+, C5+, O7+.
b) Qui luật liên hệ giữa En với Z tính được ở trên phản ánh mối liên hệ nào giữa hạt nhân với electron
trong các hệ đó ?
c) Trị số năng lượng tính được có quan hệ với năng lượng ion hoá của mỗi hệ trên hay không ? Tính
năng lượng ion hoá của mỗi hệ.
1.3. Áp dụng thuyết lai hoá giải thích kết quả của thực nghiệm xác định được BeH2, CO2 đều là phân
tử thẳng.
CÂU 2.

Hình 2.1. Hai kiểu sắp xếp nguyên tử trên một lớp.
2.1. Trong mạng hai chiều, các nguyên tử có thể sắp xếp theo kiểu A hoặc B (hình 2.1 bên trên). Hãy
tính tỉ lệ diện tích nguyên tử trên tổng diện tích trong mỗi trường hợp trên.
2.2. Từ mạng hai chiều, tiếp tục xếp các lớp nguyên tử khác lên ta thu được mạng ba chiều. Có một số
cách sắp xếp như sau:
(1) Lớp thứ nhất theo kiểu A, các lớp tiếp theo đều trùng khít như lớp thứ nhất. (Kiểu s.c)
(2) Lớp thứ nhất giống kiểu A, các nguyên tử lớp thứ hai đặt khít vào khoảng trống các nguyên tử lớp
thứ nhất. (Kiểu b.c.c)
(3) Lớp thứ nhất giống kiểu B, các lớp tiếp theo trùng khít lớp thứ nhất. (Kiểu h.p)
(4) Lớp thứ nhất giống kiểu B, lớp thứ hai đặt khít vào khoảng trống lớp thứ nhất, lớp thứ ba trùng lớp
thứ nhất. (Kiểu h.c.p)
a) Tính toán phần trăm thể tích bị chiếm bởi các nguyên tử trong các trường hợp từ (1) đến (4).
55
b) Trong trường hợp (4), có một cách sắp xếp khác của lớp thứ ba là không trùng với lớp thứ nhất thu
được mạng tinh thể kiểu f.c.c. Hãy vẽ ô mang cơ sở kiểu f.c.c và tính phần trăm thể tích bị chiếm bởi các
nguyên tử trong ô mạng cơ sở.
2.3. Tính toán tỉ lệ bán kính nguyên tử tối đa (Rmax) có thể chui vào các lỗ trống của ô mạng nguyên tử
với bán kính R trong các trường hợp sau:
a) Lỗ trống tứ diện trong kiểu mạng s.c
b) Lỗ trống bát diện trong kiểu mạng f.c.c
CÂU 3.
Cacbon-14, một đồng vị phóng xạ của cacbon, thường được sử dụng trong việc xác định tuổi các mẫu
vật khảo cổ, địa chất,… Chu kỳ bán rã của cacbon-14 là T1/2 = 5730 năm. Cacbon-14 được tạo ra từ nitơ trong
khí quyển dưới tác dụng của các tia vũ trụ. Nó có thể tồn tại trong các sinh vật và thực vật thông qua quang
hợp và chuỗi thức ăn. Hàm lượng cacbon phóng xạ trong cơ sống gần như không đổi với hoạt động của cacbon-
14 là 230 Bp trên mỗi kg cacbon. Sau khi sinh vật chết đi, sự trao đổi cacbon dừng và và hàm lượng cacbon-
14 bắt đầu giảm liên tục.
3.1. Viết các phương trình của sự hình thành và phân rã cacbon-14.
3.2. Hoạt động của cacbon phóng xạ trong một mẫu vải từ kim tự tháp Ai Cập tương ứng với 480 phân
rã/giờ trên mỗi gam cacbon. Xác định tuổi của mẫu vải trên.
Trong một kim tự tháp khác, một chất bột màu trắng đã được tìm thấy. Phân tích cho thấy nó là
phenoxymethylpenicillin (Penicililin V).

Phenoxymethylpenicillin thương mại được sản xuất bởi các vi sinh vật được nuôi cấy trong một môi trường
chứa cacbohydrat (lactose, glucose, sucrose), khoáng muối và phenoxyaxetic. Người ta quyết định xác định
hàm lượng cacbon phóng xạ để ước tính tuổi của loại bột trên. Tỉ lệ số nguyên tử 14C/12C được xác định từ các
phép đo phổ là 6,0.10-13
3.3. Các nhà khảo cổ đã ước tỉnh tuổi của loại bột này từ quy luật phân rã phóng xạ. Tuổi mẫu vật họ
thu được là gì?
3.4. Giải thích kết quả tính được trên có hợp lý không? Thực tế loại bột trên được tạo thành khi nào?
Biết rằng axit phenoxyaxetic (C6H5OCH2COOH) được tổng hợp từ dầu mỏ.
CÂU 4.
Một phản ứng dùng để luyện kẽm theo phương pháp khô là:
ZnS(r) + 3/2O2(k) → ZnO(r) + SO2(k)
4.1. Tính ∆H của phản ứng ở nhiệt độ 298K và 1350K, coi nhiệt dung của các chất không phụ thuộc
o

vào nhiệt độ ở miền nhiệt độ nghiên cứu.


4.2. Giả thiết ZnS nguyên chất. Lượng ZnS và không khí (20% O2 và 80% N2 theo thể tích) lấy đúng tỉ
lệ hợp thức bắt đầu ở 298K sẽ đạt đến nhiệt độ nào khi chỉ hấp thụ lượng nhiệt tỏa ra do phản ứng ở điều kiện
chuẩm tại 1350K (lượng nhiệt này chỉ dùng để nâng nhiệt độ các chất đầu)
Hỏi phản ứng có duy trì được không, nghĩa là không cần cung cấp nhiệt từ bên ngoài, biết rằng phản
ứng trên chỉ xảy ra ở nhiệt độ không thấp hơn 1350K?

56
4.3. Thực tế trong quặng sfalerit ngoài ZnS còn chứa SiO2. Vậy hàm lượng % của ZnS trong quặng tối
thiểu phải là bao nhiêu để phản ứng có thể tự duy trì được?
Cho biết entanpi tạo thành chuẩn của các chất ở 25oC (kJ.mol-1)
Hợp chất: ZnO(r) ZnS(r) SO2(k)
∆H f
o
-347,98 -202,92 -296,90
Nhiệt dung mol đẳng áp của các chất (J.K .mol ):
-1 -1

Hợp chất ZnS(r) ZnO(r) SO2(k) O2(k) N2(k) SiO2(r)


o
Cp 58,05 51,64 51,10 34,24 30,65 72,65
Biết MZnS = 97,42g.mol-1; MSiO2 = 60,10g.mol-1
CÂU 5.
Khi nung nóng đến nhiệt độ cao, PCl5 bị phân li theo phương trình: PCl5 (k) ⇌ PCl3 (k) + Cl2 (k)
5.1. Cho m gam PCl5 vào một bình dung tích V, đun nóng bình đến nhiệt độ T (K) để xảy ra phản ứng
phân li PCl5. Sau khi đạt tới cân bằng áp suất khí trong bình bằng p. Hãy thiết lập biểu thức của KP theo độ
phân li  và áp suất p. Thiết lập biểu thức của KC theo , m, V.
5.2. Trong thí nghiệm 1 thực hiện ở nhiệt độ T1 người ta cho 83,300 gam PCl5 vào bình dung tích V1.
Sau khi đạt tới cân bằng đo được p bằng 2,700 atm. Hỗn hợp khí trong bình có tỉ khối so với hiđro bằng 68,862.
Tính  và KP.
5.3. Trong thí nghiệm 2 giữ nguyên lượng PCl5 và nhiệt độ như ở thí nghiệm 1 nhưng thay dung tích
là V2 thì đo được áp suất cân bằng là 0,500 atm. Tính tỉ số V2/V1.
5.4. Trong thí nghiệm 3 giữ nguyên lượng PCl5 và dung tích bình V1 như ở thí nghiệm 1 nhưng hạ nhiệt
độ của bình đến T3 = 0,9 T1 thì đo được áp suất cân bằng là 1,944 atm.Tính KP và . Từ đó cho biết phản ứng
phân li PCl5 thu nhiệt hay phát nhiệt. Cho: Cl = 35,453 ; P : 30,974 ; H = 1,008 ; Các khí đều là khí lí tưởng.
CÂU 6.
Vào năm 1824 nhà hóa học Đức Friedrich Wohler đã lật đổ thuyết “lực sống”. Thuyết này cho rằng
con người không thể tổng hợp được chất hữu cơ từ chất vô cơ mà không có sự trợ giúp của thần nhưng Wohler
đã làm được. Ông đã điều chế ure từ amonixianat bằng cách nhiệt phân:
NH4OCN → H2NCONH2
Hơn 150 năm sau phản ứng đã được nghiên cứu cẩn thận hơn bằng các phương pháp động học. Các dữ
kiện cho dưới đây sẽ cho biết thời gian phản ứng. Thí nghiệm bắt đầu từ lúc hòa tan 30,0g amonixianat trong
1,00 lít nước.
t (ph) 0 20 50 65 150
mure (g) 0 9,40 15,9 17,9 23,2
6.1. Tính nồng độ của amonixianat ở từng thời điểm trên
6.2. Chứng mịnh phản ứng là bậc 2 và tính hằng số tốc độ k.
6.3. Khối lượng của amonixianat còn lại là bao nhiêu sau 30 phút.
Enzym là những protein có hoạt tính xúc tác rất cao. Bảng dưới đây cho biết tốc độ hình thành oxy khi
tiến hành thí nghiệm chuyển hóa các nồng độ khác nhau của chất phản ứng bởi enzym:
[S] (mol/L 5.0·10-2 1.7·10-2 1.0·10-2 5.0·10-3 3.0·10-3
v (dm3/min) 1.66·10-5 1.24·10-5 9.52·10-6 6.25·10-6 4.26·10-6
6.4. Sử dụng phương pháp Lineweaver và Burke để chứng minh qúa trình trên phụ thuộc tuyến tính
vào phương trình Michaelis – Menten.
6.5. Vẽ đồ thị 1/v phụ thuộc vào 1/[S].
6.6. Tính hằng số Michaelis – Menten của phản ứng:

57
CÂU 7.
1. Dung dịch A gồm Ba(NO3)2 0,060 M và AgNO3 0,012 M.
a) Thêm từng giọt K2CrO4 vào dung dịch A cho đến dư. Có hiện tương gì xảy ra?
b) Thêm 50,0 ml K2CrO4 0,270 M vào 100,0 ml dung dịch A.
Tính nồng độ các ion trong hỗn hợp thu được.
2. Trình bày sơ đồ nhận biết và phương trình ion của các phản ứng đã xảy ra khi nhận biết các cation
trong dung dịch X gồm Ba2+, Fe2+, Pb2+, Cr3+, NO3-.
Cho: BaCrO4 + H2O ⇌ Ba2+ + HCrO4- + OH - ; K = 10-17,43
Ag2CrO4 + H2O ⇌ 2Ag+ + HCrO4- + OH - ; K = 10-19,50 ; pKa của HCrO4- bằng 6,50.
CÂU 8.
8.1. Cho biết các thế điện cực chuẩn: Eo(Cu2+/Cu) = 0,34V; Eo(Cu2+/Cu+) = 0,15V; Eo(I2/2I-) = 0,54V.
a) Hỏi tại sao người ta có thể định lượng Cu2+ trong dung dịch nước thông qua dung dịch KI? Cho biết
thêm rằng dung dịch bão hoà của CuI trong nước ở nhiệt độ thường (25oC) có nồng độ là 10-6M
b) Sử dụng tính toán để xác định xem Cu có tác dụng được với HI để giải phóng khí H2 hay không?
c) Muối Cu2SO4 có bền trong nước hay không? Giải thích.
8.2. Cho dòng điện 0,5A đi qua dung dịch muối của một axit hữu cơ trong 2 giờ. Kết quả sau quá trình
điện phân là trên catot tạo ra 3,865 gam một kim loại và trên anot có khí etan và khí cacbonic thoát ra.
a) Cho biết muối của kim loại nào bị điện phân? Biết rằng 5,18 gam của kim loại đó đẩy được 1,59
gam Cu từ dung dịch đồng sunfat.
b) Cho biết muối của axit hữu cơ nào bị điện phân?
c) Viết các phương trình phản ứng xảy ra trên các điện cực.
CÂU 9.
Một cô nghiên cứu sinh nhận một lô hàng gồm các halognua kim loại kiềm nhưng các bình đều mất
nhãn trừ một bình chứa kali bromua. Phòng thí nghiệm nơi cô làm việc không hề có bất kỳ loại phổ kế nào vì
vậy cô dùng cột trao đổi ion để nhận biết các mẫu halogenua kim loại kiềm đã mất nhãn . Loại nhựa cô chọn
là loại nhựa polystiren mạng lưới kiểu axit mạnh, chứa các nhóm axit sunfonic (-SO3H) nên chỉ các proton có
thể trao đổi. Cô phân tích cả sáu mẫu kim loại kiềm (và cả KBr để kiểm chứng phương pháp) theo cách sau:
Cô cân 5,00 0,01g mỗi mẫu, rồi hoà tan với nước cất trong ống đong 100mL. Cho 40mL mỗi dung
dịch qua cột; dung dịch rửa được thu vào ống đong có thể tích 250mL, rửa cột hai lần với nước cất; dung dịch
rửa này được thêm nước để được 250mL. Trước khi mẫu kế tiếp được cho vào cột, cô tái tạo proton cho nhựa
trong cột bằng cách rửa với lượng cần thiết HCl 1M rồi với nước cất. Cô chuẩn độ các mẫu 50mL của mỗi
dung dịch rửa, làm ba lần với dung dịch NaOH (nồng độ lý thuyết 3,26.10-2M) dùng chất chỉ thị là
phenolphtalein thu được các kết qủa sau:
Mẫu thí nghiệm Thể tích chuẩn độ trung bình
A 21,15  0,1mL
B 29,30  0,1mL
C 7,40  0,1mL
D 21,20  0,1mL
E 10,30  0,1mL
F 29,15  0,1mL
KBr 10,25  0,1mL
Để phân tích các kết qủa này ta có thể giả thiết rằng:
*Mỗi mẫu thử đạt >99% tinh khiết.
*Mỗi bình đều đậy chặt, không bị nhiễm nước và không khí
*Không có trường hợp hai bình chứa cùng một kim loại kiềm halogenua; lô hóa chất chỉ gồm
florua, clorua, bromua và iodua, không có hợp chất của atatin.

58
9.1. Hãy cho biết lý do vì sao phải tiến hành các thủ tục đã nêu trên? Viết phương trình phản ứng hoá
học của bất kỳ phản ứng nào đã xảy ra.
9.2. Mẫu thử nào có thể chắc chắn được nhận biết từ sự phân tích này? Mẫu thử nào có thể giới hạn kết
qủa chỉ còn hai hoặc ba khả năng?
9.3. Dùng các dụng cụ có trong phòng thí nghiệm: kính thủy tinh, giấy qùy, dung dịch natri pesunfat
(Na2S2O8) trong môi trường axit và một lọ chứa dung dịch hồ tinh bột – cô đã có thể nhận biết được cả sáu
mẫu thử. Không cần biết kết qủa thí nghiệm của cô với các hóa chất nêu trên, hãy giải thích làm thế nào với
các vật liệu trên là đủ để nhận biết được tất cả các mẫu thử chưa nhận biết được ở câu 9.2.
9.4. Tính chất nào của kim loại kiềm halogenua ngăn cản không thể nhận biết rõ ràng một số mẫu nhận
biết bằng kỹ thuật trao đổi ion dùng ở đây? Liệu một hiệu ứng như thế có phải là trở ngại đáng kể trong một
nỗ lực tương tự để nhận biết một số halogenua của kim loại kiềm thổ MX2 chẳng hạn?
CÂU 10.
10.1. So sánh nhiệt độ sôi của các chất sau và giải thích ngắn gọn:

10.2. Benzene và ethylbenzene có giá trị nhiệt độ sôi và nhiệt độ nóng chảy như sau (không theo thứ
tự):
Nhiệt độ sôi: 80oC và 136oC, nhiệt độ nóng chảy: -95oC và 5oC. Hãy sắp xếp nhiệt độ sôi và nhiệt nóng
chảy tương ứng với mỗi chất và giải thích ngắn gọn.
10.3. Cho giá trị moment lưỡng cực của hai hợp chất sau: CH3F (1,84D) và CD3F (1,85D). Từ dữ kiện
thực nghiệm trên, hãy so sánh độ âm điện của hydrogen và deuterium? Đề xuất phương án giải thích nguyên
nhân sự chênh lệch độ âm điện giữa hydrogen và deuterium.
10.4. So sánh độ dài liên kết CO trong các phân tử sau và giải thích ngắn gọn:

Chú ý: phân tử B3 có 2 liên kết C-O.


10.5. So sánh tính acid của các nguyên tử hydrogen (được in đậm) sau đây và giải thích ngắn gọn:

10.6. So sánh tính acid của các nguyên tử hydrogen (được in đậm) trong phân tử hydrocarbon sau đây
và giải thích ngắn gọn:

**********HẾT**********

59
SỞ GD VÀ ĐT QUẢNG TRỊ ĐỀ THI ĐỀ NGHỊ DUYÊN HẢI BẮC BỘ NĂM 2022
TRƯỜNG THPT CHUYÊN LÊ QUÝ ĐÔN MÔN: HOÁ HỌC LỚP 10
Thời gian làm bài: 180 phút
----------------------------

Câu 1. (2,0 điểm) Cấu tạo nguyên tử, phân tử, định luật tuần hoàn
1) Ở nhiệt độ rất cao, nguyên tử oxi có thể bị ion hóa và tồn tại dưới dạng ion O7+. Dựa vào công thức tính
năng lượng electron của Bohr:
Z2
E n = −13,6 (eV)
n2
Hãy tính bước sóng của bức xạ phát ra khi electron trong ion O7+ dịch chuyển từ mức năng lượng có
n = 3 xuống mức có n = 1.
2) Sắp xếp theo trật tự tăng dần, có giải thích ngắn gọn nhiệt độ nóng chảy của: MgO, FeCl3, KCl, NaCl.
3) Cho số liệu về năng lượng liên kết và độ dài liên kết của các phân tử, ion như sau:
N2 N +2 O2 O+2
Năng lượng liên kết (kJ.mol‒1) 945 841 498 623
Độ dài liên kết (pm) 110 112 121 112
Dữ kiện trên cho thấy khi N2 mất đi một electron để hình thành ion N +2 thì năng lượng liên kết giảm, độ
dài liên kết tăng; trong khi đó, khi O2 mất đi một electron để hình thành ion O+2 thì năng lượng liên kết lại
tăng còn độ dài liên kết giảm. Dựa vào thuyết MO hãy giải thích hiện tượng này.
Câu 2. (2,0 điểm) Tinh thể
1) Khối lượng riêng của barium vào khoảng 3,65 g/cm3 và bán kính nguyên tử của nó là 217,4 pm.
Cho biết cấu trúc tinh thể barium, biết nó kết tinh ở một trong các kiểu mạng lập phương.
2) Ái lực electron thứ hai của O không thể được xác định trực tiếp trong phase khí ( O− + e → O2− ) . Sử dụng
dữ liệu dưới đây và năng lượng mạng lưới tính được ở (1) để xây dựng chu trình Born-Haber và tính giá trị
này.
O-(g) → O(g) + e 141,8 kJ.mol-1
O2(g) → 2O(g) 498,4 kJ.mol-1
Mg(s) → Mg (g) 146,4 kJ.mol-1
Mg(g) → Mg+(g) + e 137,7 kJ.mol-1
Mg+(g) → Mg2+(g) + e 1450,6 kJ.mol-1
Mg(s) + ½ O2(g)→ MgO(s) -601,2 kJ.mol-1
Mg2+(g) + O2-(g) → MgO(s) - 3943kJ.mol-1
Câu 3. (2,0 điểm) Phản ứng hạt nhân
1) Technetium-99 được sử dụng làm tác nhân phóng xạ trong xét nghiệm quét xương (99Tc43 được hấp thụ vào
xương). Nếu 99Tc có chu kì bán rã là 6,0 giờ thì còn bao nhiêu phần của liều dùng 100 μg 99Tc còn lại trong cơ
thể bệnh nhân sau 2 ngày?
2) Tuổi của mẫu đá mà con tàu Apollo 16 lấy từ mặt trăng đã được xác định từ các dữ kiện về tỉ lệ 87Rb / 86Sr
và 87Sr / 86Sr của các loại khoáng khác nhau có trong mẫu đá.

60
Khoáng 87Rb / 86Sr 87Sr/ 86Sr
A (Plagioclase) 0,004 0,699
B (Quintessence) 0,180 0,709
a) Rb phân rã , viết phản ứng phân rã này. Chu kỳ bán hủy của phân rã này là 4,8 × 1010 năm.
87

b) Tính tuổi của mẫu đá. Bạn có thể giả sử rằng tỉ lệ 87Sr / 86Sr ban đầu là như nhau trong cả A và B, 87Sr
lẫn 86Sr đều bền vững.

Câu 4. (2,0 điểm) Nhiệt hóa học


1) Trộn 400 mL dung dịch một axit yếu đơn nấc 0,200 M với 100 mL dung dịch NaOH 0,800 M trong một
bình cách nhiệt. Nhiệt độ dung dịch hỗn hợp tăng lên từ 25,08oC đến 26,25oC khi kết thúc phản ứng. Tính hiệu
ứng nhiệt của phản ứng trung hòa trên. Giả thiết khối lượng riêng của tất cả các dung dịch là 1,00 g/cm3 và nhiệt
dung của chúng đều là 4,2 J/g∙K.
2) Xét quá trình hóa hơi 1 mol nước lỏng ở 50oC và 1 atm. Cho biết nhiệt dung đẳng áp của hơi nước, của
nước lỏng và nhiệt hóa hơi của nước lần lượt là:
Cp,H2O(l) = 75,31 J/mol.K; Cp,H2O(k) = 33,47 J/mol.K; Hhh (100oC, 1 atm) = 40,668 kJ/mol.
Các dữ kiện trên được chấp nhận coi như không đổi trong khoảng nhiệt độ khảo sát. Biết với quá trình đẳng
áp, biến thiên entropy được tính theo hệ thức: S = Cp.ln(T2/T1) (với T2 > T1).
a) Tính H, S, G của hệ trong quá trình hóa hơi nói trên.
b) Dựa vào kết quả thu được, hãy rút ra kết luận quá trình hóa hơi của nước trong điều kiện trên là thuận
nghịch hay không thuận nghịch, có thể tự diễn ra hay không? Giải thích?
Câu 5. (2,0 điểm) Cân bằng hoá học trong pha khí
Thực hiện phản ứng: 2NOCl(k) 2NO(k) + Cl2(k)
1) Ban đầu cho vào bình phản ứng NOCl, thực hiện phản ứng ở 3000C. Khi hệ đạt trạng thái cân bằng thấy
áp suất trong bình là 1,5 atm. Hiệu suất của phản ứng là 30%. Tính hằng số cân bằng của phản ứng.
2) Ở nhiệt độ 3000C, phản ứng có thể tự xảy ra được không? Vì sao?
3) Thực hiện phản ứng và duy trì áp suất của hệ phản ứng ở điều kiện đẳng áp: 5 atm. Tính phần trăm số mol
của các khí ở trạng thái cân bằng?
4) Một cách cẩn thận, cho 2,00 gam NOCl vào bình chân không có thể tích 2,00 lít. Tính áp suất trong bình
lúc cân bằng ở 3000C.
Câu 6. (2,0 điểm) Động hóa học hình thức
1) Tốc độ của phản ứng khử HCrO4 – bằng HSO3 – được biểu diễn bằng phương trình tốc độ:
V = k.[HCrO4 – ][HSO3 – ]2[H+]
Trong một thí nghiệm với các nồng độ ban đầu: HCrO4– = 10-4 mol/l; HSO3 – = 0,1 mol/l; H+ cố định bằng 10-
5
mol/l thì sau 15 giây thấy nồng độ HCrO4- giảm xuống còn 5.10-5 mol/l.
a) Sau bao lâu nồng độ HCrO4– sẽ bằng 1,25.10–5 M? Tính hằng số tốc độ của phản ứng.
b) Nếu nồng độ đầu của HSO3– là 0,01M thì sau bao lâu nồng độ của HCrO4– sẽ bằng 5.10–5 M.
c) Nếu nồng độ ban đầu của HSO3– và H+ đều bằng 10–3 M và được giữ cố định thì cần thời gian bao
lâu để một nửa lượng HCrO4 – bị khử?
2) Bằng thực nghiệm người ta thu được những số liệu của phản ứng giữa NO và H2 ở nhiệt độ 7000C như sau:
2NO(k) + 2H2(k) ⎯⎯ → 2H2O(k) + N2(k)
Thí nghiệm [H2], M [NO], M Tốc độ ban đầu, M.s-1
1 0,010 0,025 v1=2,4.10-6
61
2 0,0050 0,025 v2=1,2.10-6
3 0,010 0,0125 v3=0,60.10-6
Xác định phương trình động học và bậc của phản ứng.
Câu 7. (2,0 điểm) Dung dịch và phản ứng trong dung dịch
Trong dung dịch chứa ion Zn2+ và ion C2O42- có thể có các cân bằng sau:
Zn2+ + C2O42- ⇌ ZnC2O4* β1 = 104,85
Zn2+ + 2C2O42- ⇌ Zn(C2O4)22- β2 = 107,55
Zn2+ + C2O42- ⇌ ZnC2O4 (r) Ks-1 = 107,56
Trong đó ZnC2O4* là dạng phức tan trong nước còn ZnC2O4 (r) là kết tủa.
1) Bằng tính toán hãy cho biết có thể kết tủa được hoàn toàn ion Zn2+ từ dung dịch Zn2+0,10 M bằng dung
dịch C2O42- hay không? Nếu được, cho biết nồng độ C2O42-cần sử dụng.
2) Tính pH của dung dịch nước để độ tan của ZnC2O4 là 0,01 M (thu được dung dịch B).
3) Nhỏ 3 giọt H2S 0,100 M (thể tích là 0,10 mL) vào 10,0 mL dung dịch B (coi thể thích dung dịch không
thay đổi). Có hiện tượng gì xảy ra hay không?
Cho biết: H2C2O4 có pKa1 = 1,25; pKa2 = 4,27; H2S có pKa1 = 7,02; pKa2 = 12,90; pKs(ZnS) = 23,8.
Câu 8. (2,0 điểm) Phản ứng oxi hoá khử. Pin điện và điện phân.
Hòa tan 0,168 g đồng (II) nitrat vào nước rồi pha loãng đến 100 mL. Dung dịch thu được có pH = 4,4. Biết
E (Cu+/Cu) = 0,52 V và Eo (Cu2+/Cu+) = 0,16 V
o

1) Tại sao dung dịch đồng (II) nitrat có phản ứng axit. Tính pKa của nấc phân ly thứ nhất.
2) Chứng minh rằng Cu+ không bền trong dung dịch. Tính hằng số cân bằng của phản ứng dị phân này ở
25oC. Ở 250C: RT/nF = 0,059;
3) Ở trạng thái cơ bản ion Cu(II) hay Cu(I) bền hơn? Điều này có thay đổi trong dung dịch nước hay không?
Giải thích.
4) Hòa tan 10 mmol CuNO3 vào 1L nước ở 25oC. Tính nồng độ của các ion đồng có mặt trong dung dịch tại
thời điểm cân bằng.
5) Cu2O được cho vào dung dịch muối Cu(II) có nồng độ Cu2+ là 0,01 M ở 22oC (Ksp (CuOH) = 1,0.10-15).
Để Cu2O tồn tại bền trong dung dịch này thì pH của dung dịch sẽ là bao nhiêu? Bên cạnh đó hãy cho biết ảnh
hưởng của nhiệt độ trong khoảng từ 0 – 100oC. F = 96500 (C)
Câu 9. (2,0 điểm) Halogen, Oxi – lưu huỳnh
1) Các hợp chất X, Y, Z đều cấu tạo gồm các nguyên tố Na, S, O trong đó MZ – MY = MY – MX = 16. Khử
Y bằng cacbon ở nhiệt độ cao rồi cho sản phẩm vào dung dịch HCl thu được một chất khí mùi trứng thối. Khí
này tác dụng với dung dịch HClO thu được sản phẩm chứa lưu huỳnh có cùng số oxi hóa với lưu huỳnh trong
Y.
Từ dung dịch X có thể trực tiếp điều chế Z bằng cách hòa tan vào Z một đơn chất, sau đó cô đặc dung dịch và
kết tinh để thu được một tinh thể ngậm 5 phân tử nước. Lọc vớt tinh thể rồi làm khô, đun nóng nhẹ được dung
dịch chứa Z với nồng độ C%.
a) Xác định X, Y, Z và viết các phương trình phản ứng minh họa.
b) Tính giá trị của C.
2) Axit flosunfuric có công thức là HSO3F là một axit mạnh. Ở trạng thái lỏng, HSO3F tự phân ly theo cân
bằng sau:
2HSO3F [HOSO2FH]+ + [SO3F]-
a) Xác định cấu trúc của HSO3F, [H2SO3F]+ và [SO3F]-
b) So sánh lực axit của HSO3F và H2SO4. Giải thích.
62
Câu 10. (2,0 điểm) Đại cương hữu cơ (quan hệ giữa cấu trúc và tính chất)
1) Saquinavir là một loại thuốc dùng để trị bệnh HIV (Human Immunodeficiency Virus - hội chứng suy giảm
miễn dịch mắc phải) có tên thương mại là Invirase, có cấu trúc như sau:

Xác định các trung tâm bất đối trong phân tử saquinavir và cho biết mỗi trung tâm đó có cấu hình R hay
S.
2) Cho 4 axit: CH3CH2COOH (A); CH3COCOOH (B); CH3COCH2COOH (C); CH3CH(+NH3)COOH
(D). Sắp xếp A, B, C, D theo trình tự tăng dần tính axit. Giải thích.
3). Cho hai chất sau đây:
O CH2

CH3
CH2
A1 A2

Giữa A1 và A2, chất nào dễ tham gia phản ứng với Br2 theo cơ chế cộng electrophin (vào liên kết đôi
C=C)? Giải thích.
-----------------------------------

63

You might also like